Complex Analysis 1 Latest (Repaired)

You might also like

Download as pdf or txt
Download as pdf or txt
You are on page 1of 117

1

UNIVERSITY OF NAIROBI

COLLEGE OF BIOLOGICAL AND PHYSICAL SCIENCES

FACULTY OF SCIENCE

In collaboration with

CENTRE FOR OPEN AND DISTANCE LEARNING

MATHEMATICS

SMA306:COMPLEX ANALYSIS 1

AUTHOR
DR: MURIUKI JOHN NYOGORE
2

UNIVERSITY OF NAIROBI

FACULTY OF SCIENCE

In collaboration with

CENTRE FOR OPEN AND DISTANCE LEARNING

MATHEMATICS

SMA 306: COMPLEX ANALYSIS 1

WRITTEN BY
DR: MURIUKI JOHN NYOGORE
SCHOOL OF MATHEMATICS
UNIVERSITY OF NAIROBI

REVIEWED BY DR:HUDSON JAMEN WERE

EDITED BY:
3

Published by the University of Nairobi, P.O. Box 30197, Nairobi, Kenya


Printed by the college of Education and External Studies, University of Nairobi,
P.O. Box 30197, Nairobi, 2010

©University of Nairobi Council, 2010, all rights reserved. No part of this module
may be reproduced in any form or by any means without permission in writing
from the Publisher.
4

COURSE OUTLINE: COMPLEX ANALYSIS I

LECTURE ONE: ARITHMETIC OPERATIONS ON COMPLEX NUMBERS


Page

1.0. Introduction………………………………………………………………………..6
1.1. Introducing complex numbers…………………………………………………….6
1.2. Geometric representation of complex numbers………………………………….10
1.3. Trigonometric form of a complex number……………………………………….15
1.4. Powers and roots of complex numbers…………………………………………..20

LECTURE TWO: FUNCTIONS OF A COMPLEX VARIABLE


2.0. Introduction………………………………………………………………………27
2.1. Functions of a complex variable…………………………………………………27
2.2. The basic elementary functions of a complex variable…………………………..31
2.3. The limit of a sequence of complex numbers……………………………………35
2.4. The limit and continuity of a function of a complex variable……………………39

LECTURE THREE: ANALYTIC FUNCTIONS


3.0. Introduction………………………………………………………………………45
3.1 Complex differentiation………………………………………………………….46
3.2. The Cauchy-Riemann equations…………………………………………………52
3.3. Harmonic functions………………………………………………………………59

LECTURE FOUR: INTEGRATING FUNCTIONS OF A COMPLEX VARIABLE


4.0. Introduction………………………………………………………………………62
4.1. Simply and multiply connected regions………………………………………….63
4.2. Conventions regarding the traversal of a curve………………………………….64
4.3. Connection between real and line integral……………………………………….64
4.4. Green’s theorem in the plane…………………………………………………….70
4.5. Cauchy’s theorem………………………………………………………………..74
4.6. Cauchy’s integral formulas……………………………………………………... 77

LECTURE FIVE: POWER, TAYLOR AND LAURENT SERIES IN THE COMPLEX


DOMAIN
5.0. Introduction………………………………………………………………………81
5.1. Definitions and convergence tests……………………………………………….81
5.2. Power series and radius of convergence…………………………………………82
5.3. Taylor and Laurent series……………………………………………………….. 84

LECTURE SIX: ZEROS OF A FUNCTION AND ISOLATED SINGULARITIES


6.0. Introduction………………………………………………………………………91
6.1. Zeros of a function……………………………………………………………….91
6.2. Isolated singular points…………………………………………………………. 93
5

LECTURE SEVEN: RESIDUES OF A FUNCTION


7.0. Introduction………………………………………………………………………95
7.1. Definitions and examples………………………………………………………..96
7.2. The residue theorem…………………………………………………………….100
7.3. Applying residues to evaluate definite integrals………………………………. 100
6

INTRODUCTION TO THE COURSE

In this module we start by surveying the algebraic and geometric structures of the
complex number system. W e assume various corresponding properties of real numbers
to be known. We then go ahead and consider functions of a complex variable and develop
a theory of differentiation for them. The main goal of the module is to introduce analytic
functions, which play a central role in complex analysis. The first objective of the module
is to develop those parts of theory which are prominent in applications of the subject. The
second objective is to furnish an introduction to applications of residues.
The module will have a total of seven lectures. In lecture one; you will be introduced to
the basic ideas and definitions of a complex number that forms the basis of the study of
functions of a complex variable.
Lecture two brings you the introduction of functions of a complex variable. In lecture
three, we proceed to the concept of analytical functions. Lecture four introduces the
concept of integrating a function of a complex variable. Lecture five is on power series,
Taylor and Laurent series in the complex domain. Lecture six gives the definitions and
ideas on zeros of a function and isolated singularities. The last lecture number seven is
devoted to the concepts of residues of a function which have wide applications in
different scientific fields.
Each lecture is divided into various sections beginning with the general introduction of
the lecture. Within each section there are provided worked out examples, followed by
exercises for you to do based on the worked out problems. You are required to go
through all these exercises before proceeding to the next part.

In order to asses your progress, a self-test is given at the end of each lecture. Answers to
the self test exercises will be given at the very end of the lecture but please do not look at
the answers before attempting the questions.
Wish you all the best as you go through the module.

TAKE NOTE

You are given some exercises in the form of activities at the end of each lecture.
These activities are intended to help you understand the concepts already discussed.
You should therefore take them seriously and try to work through all of them
7

GENERAL OBJECTIVES

At the end of this module you should be able to:


(i) Give some of the basic properties associated with complex numbers
(ii) Prove properties associated with the concepts of limits, continuity and to evaluate
problems associated with such properties
(iii) Determine whether or not a function is analytic and also be able to solve problems
associated with harmonic functions
(iv) Prove some of the properties of the elementary functions and also to work out some
simple relations between them
(v) Prove Cauchy’s theorem, use this theorem to evaluate simple integrals, and be able to
determine by way of examples the property of independent path
(vi) Derive Cauchy’s integral formulas and use them to evaluate given integrals and also be
able to determine bounds for given analytic functions
(vii) Carry out both Taylor and Laurent series expansion of given analytic functions
(viii) Carry out a Laurent series expansion of a given function which may fail to be analytic at
some points
(Viiii) Classify and obtain points of singularities of given functions
(xi) Define and calculate residue, evaluate given integrals using the method of residues,
evaluate certain real integrals

LECTURE ONE: ARITHMETIC OPERATIONS ON COMPLEX NUMBERS.

1.0. INTRODUCTION

Welcome to the first lecture in this module. Here we are going to survey the algebraic
and geometric structure of the complex number system. We introduce you to the system
of complex numbers as an extension of the familiar real number system. One of the most
striking differences between the real numbers and the complex numbers is the fact that
complex numbers have a two-dimensional character , whereas real numbers are
essentially one –dimensional. This two –dimensional aspect of complex numbers leads to
a most useful representation of them as points in a plane. This generalization will allow
us (for example) to find then square root of negative numbers.
Complex numbers are useful in many applications of mathematics. Electrical
engineering, fluid dynamics are two areas of study whose development has been
facilitated by the use of complex numbers.
8

OBJECTIVES

At the end of this lecture you should be able to:


(i) Define a complex number
(ii) Perform the basic operations on complex numbers
(iii) Represent a complex number in polar, exponential forms and associated properties
(iv) Evaluate powers or roots of complex numbers
(v) Show the relations between point sets in the complex plane

The lecture has three major aims: to give you an idea of how and why complex numbers
arose, to enable you to manipulate them, and to illustrate some of their applications.
If you have studied SMA 101, Basic mathematics, then most of the material in this
lecture should already be familiar to you.

1.1. INTRODUCING COMPLEX NUMBERS

Historically one of the first problem in which the square root of a negative number
occurred was posed by the algebraist DIOPHANTUS OF ALEXANDRIA.
He was trying to find the dimension of a right-angled triangle which would have area 7
and perimeter 12.

x2  y 2
x
Area=7
Perimeter=12

y
9

If x and y are the length of the two sides adjacent to the right angle then the hypotenuse
1
has the length x 2  y 2 . As the area of the triangle is 7, we have xy  7 .
2
From the perimeter being 12, we obtain the equation x  y  x  y  12
2 2

Re-arranging this last equation in the form x 2  y 2  12  x  y and then


squaring both sides to eliminate, the square root, we obtain
x2  y 2  144  x2  y 2  24 x  24 y  2 xy . This simplifies to
12x+12y-xy-72=0
In his attempt to solve this problem he ended up with the equation (which in modern
notation is)
6 x2  43x  84  0
This equation has roots which involve the square root of a negative quantity.
It was the German mathematician Karl Friendrich Gauss, who gave the name complex
numbers to expressions that involve the square root of negative numbers. Today the
subject of complex analysis is recognized as one of the most powerful mathematical
instruments, not only of mathematicians (pure and applied) but also of engineers,
physicists and other scientists.

Definitions 1.1.1

A complex number, usually denoted by z is completely specified by a pair of real


numbers x and y:
z  x  iy
(The algebraic form of a complex number), where i  1 ( i is known as the
2

imaginary unit).
The number x is called the real part of the complex number and y the imaginary part.
The notations are x=Rez , y=Imz.

Definitions 1.1.2

z  x  iy we associate the complex conjugate of z


With every complex number

z  x  iy . Hence Re z  Re z and Im z   Im z .
Some basic properties which are associated with this definition may be stated as follows:
10

Theorem 1.1.3
Let z  , andz its conjugate, then
(i) zz  x  y2 2

z z z x  iy
(ii) z 1   2 2 
zz z x  y 2 x2  y 2
The real part Rez and the imaginary part Imz of a complex number z can be expressed in
z thus:
terms of z and
zz zz zz
Re z  , Im z  i 
2 2 2i
Proof

Let z  x  iy and z  x  iy . On adding z and z , we obtain


zz zz
z  z  2x  x  but x  Re z 
2 2
On subtracting z from z , we obtain
zz zz
z  z  2iy  y  i .
2i 2
Example 1.1.4

Show that z1  z2  z1  z2
Proof
By definition
z1  z2   x1  x2   i  y1  y2    x1  iy1    x2  iy2   z1  z2
In a very similar approach, attempt the following exercise
11

ACTIVITY
EXERCISE 1.1.5
Prove that
(i) z1  z2  z1  z2 (ii) z1 z2  z1 z2

 z1  z1
 
z   z
(iii) (vi)
 z2 1.1.3
DEFINITION  z2

Two complex numbers z1  x1  iy1 and z2  x2  iy2 are assumed to be


equal if and only if x1  x2 and y1  y2 i.e. if the real parts are equal
and their imaginary parts are equal.

Example 1.1.6

Find the real solutions of the equation  4  2i  x   5  3i  y  13  i


Solution

Separate the real and imaginary parts of the left-hand side of the equation i.e.
write  4 x  5 y   i  2 x  3 y   13  i
Thus according to the condition of equality of two complex numbers, we get
4 x  5 y  13

2 x  3 y  1
To solve this system of equations, we can use elimination method:
Multiply the second equation by 2 and subtract it from equation one to
obtain:
4 x  5 y  13

4 x  6 y  2
12

11y  11  y  1 .Substitute y  1 into equation one: 4x  5  13  x  2 i.e


Solving this system of simultaneous equations, we find that
x=2 ,y=1
In a very similar manner to the one above do the following exercises

ACTIVITY
EXERCISE 1.1.7

1. Find the real solutions of the following equations


(i)  3x  i  2  i    x  iy 1  2i   5  6i
(ii)  x  iy  a  ib   i 5
, where a and b are given real

numbers, and a  b.
2. Perform each of the indicated operations
(i) 3  2i    7  i 
(ii)  7  i   3  2i 
3  2i
(iii)
1  i
3i 30  i10
(iv)
2i  1
3. Find the real numbers x and y such that 3x  2iy  ix  5 y  7  5i
13

1.2. GEOMETRICAL REPRESENTATION OF COMPLEX


NUMBERS

Any complex number z  x  iy can be represented in a plane XOY


(called the complex number plane, the Gauss Argand plane, or the Gaussian
plane) by either the point (x,y) ( see fig below) or the vector OM whose tail
is at point (0,0) and tip at point M(x,y).

Y
M
Fig 1


X
O

The y-axis is called the imaginary axis, or axis of imaginaries and the x-axis
is the real axis, or axis of reals.
In certain instances, it is convenient to consider the vector OM as the
geometric representation of the complex number z  x  iy .
The length of  of vector OM is known as the modulus of the complex
number and is denoted by z , so that   z  x2  y 2 .
The angle  between the vector OM and the x-axis is called the argument
of z and is denoted by  Argz .
The argument of a complex number is determined only up to an integer
multiple of 2 . Argz  arg z  2k  k  0,  1,  2,...
where argz is the principal value of Argz, namely   arg z  
with
14

  y
 arctan   if x  0
 x
  y
  arctan   if x  0, and y  0
 x
  y
arg z    arctan   if x  0 and y  0 (1.2.1)
 x

 if x  0 and y  0
2
 
 2 if x  0 and y  0

The following formulas are true:


y y
tan  Argz   , sin  Argz   ,
x x y
2 2

x
cos  Argz  
x2  y2

Example 1.2.2

Find the modulus and the argument of the complex number


 
z   sin( )  i cos( )
8 8
Solution
   
We have x   sin    0 , y   cos    0
8 8
The principal value of the argument, according to (1.2.1) is
15

       5
arg z    arctan  cot     arctan  tan       
 8   2 8  8
Hence
5
Argz     2k , k 
8
   
z  sin 2    cos 2    1
8 8
In the same way try the following numbers

ACTIVITY
EXERCISE 1.2.3

In the problems below find the modulus and the principal value of the argument of
the following complex numbers:
(i) z  4  3i

(ii) z  2  2 3i
 
(iii) z   cos  i sin
5 5
3
(iv) z  cos  i sin  (    )
2
16

Definition 1.2.4

Two complex numbers z1 and z2 are considered equal if and only if their
moduli are equal and their arguments are equal or differ by an integer
multiple of 2 .
z1  z2 , Argz1  Argz2  2 n  n=0,
 1,  2, ..
Suppose we have two complex numbers z1  x1  iy1 and z2  x2  iy2
1. The sum z1 + z2 of z1 and z2 is a complex number defined thus
z1 + z2 =  x1  x2   i  y1  y2 
2. The difference z1 - z2 of z1 and z2 is defined as
z1 - z2 =  x1  x2   i  y1  y2 
3. The product z1 z2 of z1 and z2 is given by the following formula:
z1 z2   x1 x2  y1 y2   i  x1 y2  x2 y1 
z z  x2  y 2  z
2
This definition yields, for one
z1
4. The quotient of the division of z1 by z2 z 2
 0  is a complex
z2
z1
number Z such that zz2  z1 . For we have
z2
z1 z1 z
= 2 (1.2.5)
z2 z2
z2
Here we used the formula z 2
1
 2 . We can also write (1.2.5) as
z2
z1 x1 x2  y1 y2 x2 y1  x1 y2
=  i
z2 x22  y22 x22  y22
17

1.3. TRIGONOMETRIC FORM OF A COMPLEX NUMBER

A(a,b)

Fig. 2
r
b


x
0 a

Denote by  and r  r  0  the polar coordinates of the point


A (a,b)
and consider the origin of the
pole and the positive direction of the x-axis, the polar axis. Then (fig 2) we
have a  r cos , b  r sin 
and hence the complex number may be given in the form
a  ib  r cos  ir sin  or z  r  cos   i sin   (1.3.1)
The expression on the right is called the trigonometric form (or polar form)
of the complex number z  a  ib ; r is termed the modulus of the
complex number Z,  is the argument ( amplitude or phase) of the
complex number Z. They are designated as r  z ,   arg z (1.3.2)
The quantities r  r  0  and  are expressed in terms of a and b as
follows:
b
r  a 2  b2 ,   Arc tan
a
In summary we have:
18

z  a  ib  a 2  b 2
b (1.3.3)
arg z  arg  a  ib   Arg  
a
It will be noted that the real number A can also be written in the form
A  A  cos0  i sin 0  For A>0
A  A  cos   i sin   For A<0
The modulus of the complex number 0 is zero, z  0.
Any angle  may be taken for amplitude zero. Indeed, for any angle 
we have 0  0  cos   i sin  

Example 1.3.4
Write the complex number z  1  i 3 in polar form

Solution

 1   3  
2
z  2
2
We have ;

 3
tan    3
1
2
Hence    
3
 2  2 
Therefore z  1  i 3  2 cos   
  i sin   
  3   3 

Example 1.3.5

Similarly do the following numbers

ACTIVITY
EXERCISE 1.3.6
19

Let the complex numbers be written in trigonometric form


z1  r1  cos 1  i sin 1  , z2  r2  cos  2  i sin  2  , then

(a) z1 z2 = r1  cos 1  i sin 1  r2  cos  2  i sin  2 

=
rr 
1 2 
cos 1
cos  2
 i sin 1
cos  2
 i cos 1
sin  2
 i 2
sin 1 sin 2 

=
rr cos1 cos2  sin 1 sin 2  i  sin 1 cos 2  cos 1 sin 2 
1 2 

= rr cos
1 2       i sin    
1 2 1 2
then
20

z1 z2  rr cos 1  2   i  sin(1  2 ) 


1 2 
(1.3.7)
i.e. the product of two complex numbers is a complex number, the modulus
of which is equal to the product of the moduli of the factors, and the
amplitude is equal to the sum of the amplitude of the factors.
z1 z2  z1 z2

Arg  z1 z2   Argz1  Argz2

z1 r1  cos 1  i sin 1 
(b) 
z2 r2  cos  2  i sin  2 

r1
cos 1   2 )  i sin(1   2  
r2 
= (1.3.8)

To verify the equation, multiply the divisor by the quotient:


r1
r2  cos  2  i sin  2  cos 1   2   i sin 1   2   
r2
r1
r2 cos  2  1   2    i sin  2  1   2   r1  cos 1  i sin 1 
r2
z z1 z
i.e. 1  , Arg 1  Arg  z1   Arg  z2 
z2 z2 z2
Thus, the modulus of the quotient of two complex numbers is equal to the
dividend and the divisor; the amplitude of the dividend and the divisor.

Example 1.3.9

Find the product


            
2 cos    i sin    3 cos    i sin   
 6  6     12   12  
Solution
21

By using the formula given above we obtain


            
2 cos    i sin    3 cos    i sin    
 6  6     12   12  
     
 2  3 cos     i sin     
  6 12   6 12  
       2 2
6 cos    i sin     6  i   3 2 1  i 
   4  
4  2 2 
Example 1.3.10

Perform the operation of division:


  3   3        
10 cos    i sin    2 cos    i sin    
  4   4   4  4  
Solution

According to the formula given above we have


  3   3        
10 cos    i sin     2 cos    i sin    
  4   4   4  4 
10   3    3   
 cos     i sin    
2   4 4  4 4 
     
5 cos    i sin     5  0  i   5i
 2  2 
Using the same ideas as shown above, perform the indicated operations
22

ACTIVITY
Exercise 1.3.11

Perform the operation of multiplication, using the trigonometric form of the complex

number:

(i) 1  i 3  2  2i 3 
(ii) 1  i   3  3i 3 
     
(iii) 3 cos     i sin     3  i 3 
  8  8 
(iv) 3 
3  3i 2  i 2 3 
2
 1 3
(v)    i 
 2 2 

1.4. POWERS AND ROOTS OF COMPLEX NUMBERS

(A) Powers
From the formula (1.3.1) of the proceeding section it follows that if n is a
positive integer, then
r  cos  i sin    r n  cos n  i sin n 
n

This is formula is called De Moivre’s formula. It shows that when a complex


number is raised to a positive integral power, the modulus is raised to this
power, and the amplitude is multiplied by the exponent.
Now consider another application of De Moivre’s formula:
Setting r  1 in this formula, we get
 cos  i sin    cos n  i sin n
n
(1.4.1)
23

Expanding the left-hand side by the binomial theorem and equating the real
and imaginary parts, we can express sin n and cosn in terms of
powers of sin  and cos . For instance if n  3 we have
cos3   3i cos2  sin   3cos sin 2   i sin 3   cos3  i sin3
Making use of condition of equality of two complex numbers, we get
cos3  cos3   3cos  sin 2 
sin 3   sin 3   3cos 2  sin 
Example 1.4.2

 
60
Evaluate 1  i 3

Solution
Representz  1  i 3 in polar form
 2 2 
1  i 3  2  cos  i sin 
 3 3 
Applying the formula (1.4.1) for raising to a power yields
  2   2  
 1  i 3 
60
 260 cos  60   i sin  60  
  3   3 
 260  cos 40  i sin 40   260

Example 1.4.3

Applying DeMoivre’s formulas prove the validity of the following identities:


cos 2  cos 2   sin 2 
sin 2  2sin  cos 

Setting in the relationship (1.4.1) r  1 and n  2 , we get


 cos  i sin    cos 2  i sin 2 or
2
24

cos2   2i cos sin   sin 2   cos2  i sin 2


From the condition of equality of two complex numbers it follows that
cos 2  cos 2   sin 2 
sin 2  2sin  cos 
Following the procedure used for the above example, answer the following
question

ACTIVITY
Exercise 1.4.4
1. Using the DeMoivre’s formula express in terms of sin 
and cos 
the following
functions of integer multiple of :
(i) sin 3
(ii) cos3
(iii) sin 4
(iv) cos 4
2. Raise to the indicated powers:
6
 3 1    
8
 
  i  
2 cos  i sin 
8  
(i) (ii)
 2 2    8
3. Use DeMoivre’s formula to evaluate
10
 1 i 3 
 4  i4 3 
5
(i)   (ii)
 1 i 3 
25

(B) Roots

The nth root of a complex number is another complex number whose power is equal to
the radicand, or
n r  cos  i sin      cos  i sin  if

 n  cos n  i sin n   r  cos   i sin  


Since the moduli of equal complex numbers must be equal, while their amplitudes may
differ by a multiple of 2 , we have
 n  r , n    2k
Whence we find
  2k
nr ,   where k is an integer
n
n
r is the principal (positive) root of the positive number r . Therefore

   2k   2k 
r  cos    i sin   n r cos  i sin
n 
n

 n
(1.4.5)
Giving k 0,1,2,3,....., n  1 , we get n
the values different values of the root.
For the other values of k , the amplitude will differ from those obtained by a multiple of
2 , and for this reason, root values will be obtained that coincide with those
considered. Thus the nth root of a complex number has n different values.
The nth root of a real nonzero number A also has n values, since a real number is a
special case of a complex number and may be represented in trigonometric form:
If A  0 , then A  A  cos0  i sin 0 
If A  0 , then A  A  cos   i sin  
Example 1.4.6

Find all the values of the cube root of unity.


26

Solution

We represent unity in trigonometric form


1  cos0  i sin 0 . By formula (1.4.3) we have
0  2k 0  2k
3
1  3 cos 0  i sin 0  cos  i sin
3 3
k equal to 0,1,2 , we find three values of the root:
Setting
x1  cos 0  i sin 0  1
2 2
x2  cos  i sin
3 3

2 1 2 3 4 1
Noting that   ; sin
cos  ; cos 
3 2 3 2 3 2
4 3
sin  we get
3 2
1 3 1 3
x1  1, x2    i , x3    i
2 2 2 2
Example 1.4.7

Find all values of


4
1 i
Solution
1  i to polar form:
Reduce the complex number
     
1  i  2 cos     i sin     , Hence
  4  4 
   
   2 k    2k 
1  i  2  cos 4  i sin 4

4 8

 4 4 
 
Putting k  0,1,2,3 , we obtain the following results:
27

   
 k  0 4
1  i  8 2  cos  i sin 
 16 16 

 7 7 
 k  1 4
1  i  8 2  cos  i sin 
 16 16 

 15 15 
 k  2 4
1  i  8 2  cos  i sin 
 16 16 

 23 23 
 k  3 4
1  i  8 2  cos  i sin 
 16 16 
To confirm that you have understood how to compute roots of complex numbers
attempt the following exercise:

ACTIVITY
Exercise 1.4.8

In the following exercises, find all the values of the roots:

(i)
4
1 (ii) i (iii)
3
i (iv) 2  2i 3

(v)
3
1  i (vi) 4
i

As a summary of this part of the complex numbers i would suggest that you try to work
out the following mixed exercises: The answers to some selected numbers will be
provided at the end of the module.
28

ACTIVITY
Exercise 1.4.9

1  w2  iw
1. Prove that i ( w is a real number)
w  i 1 w 2

2. Prove that the polynomial

f  x    cos   x sin    cos n  x sin n x2  1


n
is divisible by
3. Specify the curves represented by the following equations:
(i) Im z 2  2 (ii) Im z 2  2 (iii) z 2  z 2  1
4. Write the equation of a straight line Ax  By  C  0 in complex
form
5. What line in the complex number plane is given by the equation?

 z  z   i  z  z   2  0
6. Write the equation of the circle x2  y 2  2x  2 y  0
in complex form
7. Evaluate
40
1 i 3 
 
6
(i)   (ii) 3  3i
 1  i 

LECTURE 2: FUNCTIONS OF A COMPLEX VARIABLE

2.0. INTRODUCTION

This lecture may be regarded as a prelude to lecture 3 on analytic functions. In this


lecture, we give definitions and provide a brief survey on some of the properties
associated with functions of a complex variable.
29

OBJECTIVES

By the end of this lecture we expect you to be able to:

a) Prove the properties associated with the concept of limits, continuity

b) Evaluate any problem associated with such properties

2.1. FUNCTIONS OF A COMPLEX VARIABLE

Recall that a function f A to a set B , i.e. f : A  B


on a set
is a rule which assigns to each element in A a unique ( i.e. one and only one) element in
B . If z and w are variables in A and B respectively, then the expression
w  f  z  is used to indicate that f is a function on A to B .
Definition 2.1.1
We say that a function w  f  z  is defined in a domain D if to each point zD
there corresponds one (simple valued function) or several ( a multiple valued function)
value of w .
We can also say that the function w  f  z  maps the points in the complex z -plane
into w-complex plane.

Z-plane B w-plane
v
y
30

w
A
z
u
x
f

To show that the definition of a function of a complex variable is clear, try and give
answers to the questions raised below:

1. Let f  z   z  2  z  . Find the values of w corresponding to:


(i) z  1  i (ii) z  2  2i
(iii) and graph the corresponding values in the z and w -plane

1 z
2. Given that w  f  z  . Find
1 z
(i) f i  (ii) f 1  i  and represent graphically

Now for z  x  iy the complex-valued function f  z  can be viewed as a


function of the complex variable z , or as a function of two real variables x and y .

The domain and range of definition of f  z  are subsets of the set of complex numbers.

Let w denote the value of the function f at the point z , i.e. let w  f  z  .
31

Then just as z decomposes into real and imaginary parts as z  x  iy , we find that
f  z  also decomposes into real and imaginary parts u and v respectively, each of
which is a real –valued function of z or, equivalently of x and y .

Thus if z  x  iy , then w  f  z   u  x, y   iv  x, y  i.e.

w  u  x, y   iv  x, y 

Whereu  x, y   Re w and V  x, y   Im w
Thus a complex-valued function f  z  of a complex variable z is in essence a pair of
real functions of two variables

Example 2.1.2

Express w  f  z   z 3  3z 2 in the form w  u  x, y   iv  x, y 


Solution

 x  iy , then
Let z

w  z 3  3z 2   x  iy   3  x  iy 
3 2


 x 3  3xy 2  i 3x 2 y  y 3 )  3  x 2  y 2  2 xyi  
 x 3  3x 2  3 y 2  3xy 2  i  3x 2 y  6 xy  y 3 

Thus u  x, y   x3  3x 2  3 y 2  3xy 2 and

v  x, y   3x 2 y  6 xy  y 3

Example 2.1.3

z2 1
Write the function w  f  z   in the form w  u  x, y   iv  x, y 
z
Solution
32

Let z  x  iy , then

z 2  1  z  1 z  z  1 z
2

w   2
z zz z
 x  iy 2  1  x  iy   x 2  y 2  1  2 xyi  x  iy  
  
x y
2 2
x2  y 2
x 3  xy 2  2 x 2 yi  x 2 yi  iy 3  iy  2 xy 2

x2  y 2
x 3  xy 2  x x2 y  y3  y
 i  u  x, y   v  x , y 
x2  y 2 x2  y 2

x 3  xy 2  x x
u  x, y    x 
x2  y 2 x2  y 2
Here

x2 y  y3  y y
v  x, y    y 
x2  y 2 x2  y 2

Following exactly the same procedure as illustrated by the worked examples, do the
following problems

ACTIVITY
Exercise 2.1.4
Separate into real and imaginary parts of f  z  , i.e. find u  x, y 
33

and v  x, y  so that f  z   u  iv
(i) f  z   z  iz 2
(ii) f  z   2 z 2  3iz
1
(iii) f z  2 
z
z
(iv) f z 
z
(v) f  z   1  z 1  z 
(vi) z3
z i
(vii) f z  2
z 1

2.2 THE BASIC ELEMENTARY FUNCTIONS OF A COMPLEX


VARIABLE

Now with the knowledge of a function of a complex variable, we can now introduce
the calculus of functions of a complex variable. But before we do that it is important first
we introduce the definitions and simple properties of some so called basic elementary
functions of a complex variable which occur in many practical problems.

(A). POLYNOMIALS
Perhaps the most obvious form of the elementary functions are the polynomial
functions.

Definition 2.2.1
The polynomial functions are defined by
w  a0 z n  a1 z n1  ....  an1 z  an  p  z 
Where a0  0, a1 , a2 ,..., an are complex constants, and n is a positive

integer called the degree of the polynomial p  z  . Notice that if n  1, then

p  z  reduces to a polynomial of degree one i.e. p  z   a0 z  a1


Any function (or transformation) of the form w  az  b , where a and b are
constants, is called a linear transformation.
34

Example 2.2.2

The function p  z   z 3  iz 3  1  i  z 2  2 z  1
is a polynomial function and w  2 z  i is a linear transformation

(B). EXPONENTIAL FUNCTIONS

We are already familiar with the real-valued exponential function f  x   ex ,


and especially its definition and properties as shown in the units on calculus and
real analysis. In defining the complex –valued exponential function f  z   ez ,
we would like to preserve the important properties of e z when z is complex-
valued, and especially the following properties:

(a) e z1 .e z2  e z1 z2 , for all z1 , z2 

e z1
 for all z1 , z2 
z1  z2
(b) e ,
e z2
(c) e z2 k i  e z ( k  0, 1, 2,... )
i.e. e is periodic function with a period of 2
z

(C). TRIGONOMETRIC FUNCTIONS

The basic trigonometric functions of complex variable are sin z and cos z
z
sin z , cos z are related via Euler’s formulas:
The functions e ,
e  cos z  i sin z
iz

and e  cos z  i sin z


 iz

by adding the two we obtain that


eiz  e  iz
cos z  , and on subtracting the two we get the formula:
2
e  e  iz
iz

sin z 
2i
Using this form of the basic trigonometric function, we can very easily justify the
some known trigonometric identities. For example we can easily show that
cos2 z  sin 2 z  1 e.t.c
35

Note that all the formulas of trigonometry remain valid for trigonometric functions
of a complex variable, for instance cos2 z  cos2 z  sin 2 z ,
sin 2 z  2sin z cos z , e.t.c
The remaining trigonometric functions are defined by the usual relations:
sin z cos z 1
tan z  , cot z  , sec z  ,
cos z sin z cos z
1
cos ecz 
sin z
With these definitions most of the familiar real-valued trigonometric properties can
be extended to the complex plane.

(D). HYPERBOLIC FUNCTIONS

The hyperbolic functions are simple linear combinations of the exponential


function:
e z  e z e z  e z
sinh  , cosh 
2 2
sinh z cosh z
tanh z  , coth z 
cosh z sinh z
The hyperbolic and trigonometric functions are coupled through the following
relations:
sin z  i sinh  iz  , and sinh z  i sin  iz 
cos z  cosh  iz  , and cosh z  cos  iz 
tan z   tanh  iz  , and tanh z  i tan  iz 
cot z  i coth  iz  , and coth z  i cot  iz 
(E). LOGARITHMIC FUNCTION

The logarithmic function Lnz  z  0  is defined as the inverse of the


exponential functions and
Lnz  Ln z  iArgz  Ln z  i arg z  2 k , k 
The following relations are true for Logarithmic functions:
36

z
Ln  z1 z2   Ln( z1 )  Ln( z2 ) , Ln( 1 )  Ln( z1 )  Ln( z2 )
z2
Ln( z k )  kLn( z)
(F). INVERSE TRIGONOMETRIC FUNCTIONS

If the quantities z and w are related by the formula


z  sin w
then w is given by the inverse function
w  sin 1 z
An alternative notation for this function is
w  arcsin z
The remaining inverse functions are
cos1 z , tan 1 z , cosec1 z , sec1 z , cot 1 z , or
arccos z , arctan z , arccosecz , arc sec z , arc cot z
are defined in a similar manner.
They are expressed in terms of logarithmic functions:


Arc sin z  iLn iz  1  z 2 
Arc cos z  iLn  z  z2  1
i 1  iz
Arc tan z   Ln( )
2 1  iz
i  z i
Arc cot z   Ln  
2  z i 
(G). GENERAL POWER FUNCTIONS

w  z a , where a    i  , is defined by z a  ea ln z

(H). GENERAL EXPONENTIAL FUNCTION

w  a z  a  0  is defined by a z  e z ln a
37

2.3 THE LIMIT OF A SEQUENCE OF COMPLEX NUMBERS

The definition of absolute value can be used to designate the distance between two
complex numbers. The concept of distance of course plays a significant role in the
study of the notions of limits and continuity. We begin here with the concept of
limit of a sequence of complex numbers.

Definition 2.3.1

A sequence  z  of complex numbers is said to have the limit z


n 0
,

(I.e.  z  converges to z ), if for any   0 , there exists an integer N such that


n 0

zn  z0   for all n>N. In this case we write


lim
n n
z  z0 , or equivalently zn  z0 as n  
I.e. z0 is the limit of the sequence  zn  if for sufficiently large n ,  zn  stays
arbitrarily close to z0

A sequence  z n  with a limit z0 is said to converge to z0

To each  z n  there corresponds two sequences  xn  and  yn  of real numbers

i.e. zn  xn  iyn , n  1,2,3,...


The sequence zn  xn  iyn converges to z0    i  if and only if
lim
n n
x   and lim n
yn  

Example 2.3.2

Using the definition of a limit of a sequence of complex numbers, prove that the
ni
sequence of numbers zn  has z0  1
n 1
as its limit
n i
In other words, prove that lim   1
n
 n  1
Solution
38

ni
Given that zn  and that z0  1
n 1

TAKE NOTE

The idea is to use the definition from right to left

ni n  i  n  1 i  1 i  1 2
z n  z0  1     
n 1 n 1 n 1 n  1 n 1

2 2
  n 1  n  1
 
2
Here we can take  1  N    ,  >0 and consequently we deduce that

ni
lim 1
n
n 1
That is, we take an arbitrary   0 and we show that there is a term in the sequence
ni ii 2
with number N such that zn  1  1     i.e.
n 1 n 1 n 1
2 2
when n   1. This means that we can take N  N     1
 
ni
 lim 1
n
n 1
Properties of sequences of complex numbers

If lim z  a and lim


n n
  b , then
n n

(i) lim
n
 zn   n   lim z  lim
n n
  ab
n n
39

(ii) lim
n
 zn  n   (lim z )(lim
n n
 )  (a)(b)
n n

 zn  lim z a
(iii) lim    n n
 ( provided lim   0)
  n  lim 
n n n
b
n n
SUFFICIENT CONDITION FOR THE CONVERGENCE OF A SEQUENCE OF A.
COMPLEX NUMBER
Let zn   n ein where  n  zn and n  arg zn
i
If lim
n
 n   0 and lim
n
 n   0 then lim zn  0e n
0

 
i lim  n
i.e. to say in other words that lim z  lim
n n n
n e n

Example 2.3.3

Using the sufficient conditions for convergence of a complex number sequence, find the
limits in the following sequences
 1  in
zn   1   e zn  1  3i 
n
(i) (ii)
 n
Solution
 1  in 1 
Given zn   1   e , here  n  1  and  n 
 n n n
 1
lim  n  lim 1    1 ,
n n
 n
  1
lim  n  lim     lim    0   0
n n
n n
n
 1  in
 According to the sufficient condition lim 1   e  1  ei 0   e0  1
 x0
 n
Given zn  1  3i 
n
(ii)

 
n n n

zn   n  1 3
2 2
 10  0  lim10  
2 2
n

n  arg zn  arg 1  3i     arctan 3  n arctan 3


n n
40

  0  lim
n
n.arctan 3  
 lim         
n i
x0
1 3i e

and hence the limit does not exist.

In a manner similar to the one above compute the limits of the given sequences

ACTIVITY
Exercise 2.3.4
Find the limits in the following given sequences
(We give only the nth term in each sequence)
ein
(i) zn  2
n
in
(ii) zn 
n
n  2i
(iii) zn 
3n  7i
 1 
i  
(iv) zn  e  2 2n 

i
(v) zn  n sin
n

n n
(vi) zn  n cos  in sin
2 2

sinh in
(vii) zn 
n
41

2.4. THE LIMIT AND CONTINUITY OF A FUNCTION OF A COMPLEX


VARIABLE

Now, a closely related concept to that of limit of complex numbers is the idea of limit of
a complex-valued function f  z.
Definition 2.4. 1

Let f  z  be defined in some neighborhood of z0 ,except possibly at z0 itself, then a


complex number w0 is said to be the limit of f  z  as z approaches z0 ,

i.e. lim f  z   w0 ( or f  z   w0 , as z  z0 ).
z zo
 , chosen arbitrary (however small), we can
If corresponding to each positive number
find another positive number  such that f  z   w0   , whenever

0  z  z0  
Example 2.4.2

 iz  i
Prove that lim  
z 1
2 2
Solution
i
For any given   0 , we must find a   0 such that f  z  
  when
2
i iz i i 1
0  z 1   i.e. f z     z 1  z 1  
2 2 2 2 2
 z  1  2 .
It is apparent therefore that the choice of   2 would suffice for the required limit.
iz i
 lim(
z 1
) 
2 2
Example 2.4.3
42

 z 2  1
Prove that lim    2i
z i
 z i 
z 2  1  z  i  z  i 
z  i ,   z i
z i z i
 z  1
2

Hence lim    lim  z  i   2i


z i
 z  i  z i

  0 , z  i  2i   , whenever z  i  
i.e. z i  , whenever z i 
 z 2  1
Here we can take    ,   0 , therefore lim    2i
z i
 z i 
The fact that there exists a limit of f  z  as z  z0 where

f  z   u  x, y   iv  x, y  , z0  x0  iy0 is equivalent to the existence


of two limits, lim
x x
u  x, y  and lim
x x
v  x, y  with
o o
y  yo y  yo

lim
zz
f  z   lim
x x
u  x, y   i lim
x x
v  x, y 
o o o
y  yo y  yo

Limits of functions of a complex variable have the following properties


Suppose that lim f  z   A and lim g  z   B
z  zo z zo

lim  f  z   g  z   lim f  z   lim g  z   A  B


zz 
(i)
o zz zz
o o
i.e. limit is distributive over addition and subtraction
(ii) lim f  z  g  z   lim f  z  lim g  z   AB
z zo z zo z zo

f  z  lim f z A
lim g  z   0 )
z  zo
lim  
g  z  lim  
(iii) (Provided
z  zo z zo
zz
g z B
o
43

Example 2.4.4

Evaluate the following limits


(i) lim
z 1i
 z 2
 5 z  10

 z 2  1
(ii) lim  6 
z i
 z  1

 cos 2 z 
(iii) lim  
z   cosh iz  i sinh iz 
4

Solutions

Note that the techniques used here to compute the limit is basically the same used in the
calculus of a real variable, namely, how to remove the so called indeterminate forms.
This could be factorization, rationalization or direct substitution among others.
lim  z 2  5 z  10  1  i   5 1  i   10 
2

(i) z 1i

1  2i  1  5  5i  10  5  3i
(ii)

 z 2  1  z 2  1   ( z 2  1) 
lim  6   lim  2 3 3
 lim  2 
z i
 z  1  z i
  z   1 
z i

  z  1 z 4
 z 2
 1 
 1  1 1
== lim  
 4 2 
 z  z  1 i  i  1 3
z i 4 2

 cos 2 z 
(iii) lim  
z   cosh iz  i sinh iz 
4
From elementary functions of a complex variable, we know that cos z  cosh iz
and sin z  i sinh iz and therefore
44

 cos 2 z 
lim  =
z   cosh iz  i sinh iz 
4

 cos 2 z 
lim  
z   cos z  sin z 
4

 cos 2  sin 2    cos z  sin z  cos z  sin z  


 lim    lim  
z   cos z  sin z  
4  
4
z cos z sin z
  2 2
== lim cos z  sin z  cos  sin    2

z
4
4 4 2 2
Applying the same ideas as shown above, do the following exercises

ACTIVITY
Exercise 2.4.5

Evaluate the following limits


 z 2  3iz  2  sin z
(i) lim   (vi) lim
 z i 
z  i z  0 sinh iz

 e2 z  1 z2  9
(ii) lim  z  (vii) lim
z  i  e  i 
z 3i z  3i
2

  2 z  3 z  1  z 1
(iii) lim  2  (viii) lim
z 2 i
 z  3 z  4 
z  z 2

cos 2 z z 2  10 z  2
(iv) lim (viiii) lim
z
 cosh iz  i sinh iz z  2 z 2  11z  6
4

z2 1
(v) lim
z i z4 1
45

Continuity of a function of a complex variable

Closely connected with the idea of limit of a function is the concept of continuity which
we express in the following definition:

Definition 2.4.6

A function f  z  defined throughout a region R in the complex plane, is said to be


continuous at a point zo  R if and only if the following conditions are satisfied:

(i) f  zo  exists
(ii) lim f  z  exists
z  zo

(iii) lim f  z   f  zo 
z zo

In other words, f  z  is continuous at zo  R if for each   o we can find


  o such that f  z   f  zo    whenever z  z0   .
For f  z  to be continuous at zo  R , it must have a limiting value at zo  R , and

this limiting value must be f  zo  . It is important to note that a function may have a

zo  R without being continuous at that point, i.e.


limit at a point

lim f  z   wo  f  zo 
z z o

A function which is not continuous at a pointzo  R is said to be discontinuous


at zo  R , or to have a discontinuity at zo  R .

A function f  z  is continuous on a region if it is continuous at each point of R.

A continuous function of a complex variable f  z   u  x, y   iv  x, y  is

continuous at point zo  xo  iyo if its real and imaginary parts u  x, y  ,

v  x, y  are continuous at  xo , yo  in x and y simultaneously.


Example 2.4.7
46

Given the linear function w  f  z   az  b , where a and b are complex


constants, prove that at point zo the function has a limit equal to wo  azo  b

 a  0
Proof
Let   o , since
f  z   wo   az  b    azo  b   az  azo  a z  zo  

or z  zo  .
a

Here we can take    limz  zo
az  b  azo  b
a
Therefore w  f  z   az  b is continuous for every point in the domain of
definition.

Example 2.4.8

Show that f  z   z 2 is continuous for all values of z


Solution
w  f  z  is continuous for z  R if and only if lim w  0
z 0

w  f  z  z   f  z    z  z   z 2
2
Here

 2 zz   z   z 2  2 zz   z 
2 2 2
=z

 lim
z 0
w  lim
z 0
2 z 
z   z   0  f 
2
z  z 2
is continuous for
any z

In a very similar manner, you are requested to do the following exercise

ACTIVITY Exercise 2.4.9


47

Prove that the following functions are continuous in the entire complex plane

1. (i) f  z  z (ii) f  z   Im z

2. (i) f  z   ez (ii) f  z   Re z
3. Examine the continuity of the functions

2i 1  z 
(i) f z  for z  0, i
z  z  1

z  z 2  1  i  z  i 
(ii) g  z  , z  0,  i
z  z  i

LECTURE 3: ANALYTIC FUNCTIONS

3.0 Introduction.
In this lecture we generalize, and where possible extend some of the properties associated
with the concept of differentiation of real-valued functions. Of paramount significance is
the concept of analytic functions and the so-called Cauchy-Riemann equations which are
often used as necessary conditions for a function f  z  to be differentiable.

OBJECTIVES
At the end of this lecture you should be able to:
(i) Determine whether or not a function is analytic
(ii) Solve problems associated with harmonic functions

3.1 COMPLEX DIFFERENTIATION

The derivative of a complex-valued function is defined by a natural extension of the


definition in the real case. We do this as follows:
48

Let f  z  be a complex-valued function defined in a neighborhood of a point z .


Take two points z  and z  z in . We introduce the notations

w  f z  z  f z ,   z  x  iy

Definition 3.1.1

w
A function w  f  z  is differentiable if the ratio has a finite limit as z  0
z
in arbitrary manner. This limit provided the limit exists is called the derivative of

f  z  at the given point z f /  z  or w/ or


dw
and is denoted by i.e. by
dz
definition,

w  f z  z  f z    
w/  f
/

z  lim    lim 
z0  z  z0
 (3.1.2)
z  
Example 3.1.3

Find by definition the derivative of the following functions


a) f  z   z2
b) f  z   3z 2  2 z  4
z 1
c) f  z 
2z 1

Solution
By definition
49

f  z  z   f  z   z  z 
2
 z2
f /
 z   lim  lim
z 0 z z 0 z

z 2  2 z z   z   z 2
2

 lim
z 0 z

2 z z   z 
2

 lim
z 0 z

 lim  2 z  z 
z 0

i.e. f /  z   2 z i.e f /  z  exists for all z and equals 2z

Similarly for (b) and (c) which i leave to you as an exercise.

As in the case of real-valued function, we can prove the following property:

Theorem 3.1.4

Let f  z  be differentiable, then f  z  is also continuous. The converse is not


necessarily true.
Proof
Since f  z  is differentiable say at z0 then by definition f /  z0  exists, where
f  z   f  z0  f  z0  z   f  z0 
 
f / z  lim
0 z z z  z0
0
 lim
z0 z
Hence
f  z   f  z0 
lim  f  z0   f  z   lim ( )  z  z0   f /  z0   0
zz 0 zz z  z0
0

i.e. lim f  z   f  z0 
z z 0

i.e. f  z  is continuous at zo ,and since z0 is arbitrary, it follows that f  z 


is continuous for all z .The converse is not necessarily true, i.e if f  z  is
also differentiable as we can see in the following example.
50

Example 3.1.5

Let f  z   z . Then f  z  is continuous at the origin for

lim f  z   f  0  0  0 .
z0

But f  z  is not differentiable at z  0 ,because

f  z   f  0 z
 , which equals 1 if z  0 , -1 if z  0 , I if z is above real axis, -I
z 0 z
if z is below real axis, i.e

1, if z  o

 1, if z  o
f  z   f 0 z 
lim  lim 
z0 z o zo z i, if z  real axis

 i, if , z  real axis

I.e. if the limit is not unique, and so the derivative


f  z   f  0
f /  0   lim does not exist at 0.
z0 z 0

Remark

It is of interest to note that because of the similarity between the derivative


of the complex-valued function f  z  and that of the real valued function
f  x  ,all the formal differentiation rules obtained for f  x  also holds true for
f  z  .We collect these in the following:

Theorem 3.1.6

Let f  z  and g  z  be differentiable, then


51

 f  z   g  z   f /  z   g /  z  and
/
a)
 f  z   g  z   f / z  g/ z
/

b) ( f  z  g  z )/  f /  z  g  z   f  z  g /  z 
 f z  f / z g z  f z g/ z
/

c)    , g  z  0
 g  z   g  z 
2

If f  z   z n , where n is an integer then


d) f /  z   n.z n1 is differentiable at z, and f is differentiable at g  z 
If g is differentiable at z,and f is differentiable at g  z  , and suppose
H  z   f  g  z   , then
e) H /  z   f /  g  z  g /  z 

Proof

These properties can infact be proved, word for word as in the case of real-
valued function and are therefore left as an exercise.

Example 3.1.7

z3  2z
Find the derivative of f  z  
z 1
Solution

On using theorem 3.1.6, we have

d d  z3  2z  
z  1
d 3

z  2z  z3 2z
d
 
 z  1 
f  z   
dz dz
dz  z  1   z  1
2
dz

 z  1  3z 2  2    z 3  2 z  2 z 3  3z 2  2
= 
 z  1  z  1
2 2

Notice that the right-hand side exists provided that z  1 ,i.e f  z  is


differentiable everywhere except at z  1 where f /  z  fails to exist,ie f / 1
is not finite.
52

Definition 3.1.8

A complex –valued function f  z  is said to be analytic (or regular or


holomorphic) if f  z  is differentiable everywhere in its domain of the
function.

Remark

We emphasize that analyticity is a property defined over open sets while


differentiability could possibly hold at one point only.
However we may still find it necessary to use the abbreviated phrase ‘ f  z 
is analytic at the point z0 '' to mean that f  z  is analytic in some
neighborhood of z0 .
Some times the function f  z  is given in terms of x and y, i.e
f  z   u  x, y   iv  x, y  in which it may not be easy to apply the above
definition to determine if f  z  is analytic. In this case it may be possible to
establish the analyticity of f  z  by making the substitution:
zz zz
x , y 3.1.9
2 2i

In the expression u  x, y   iv  x, y 

Example 3.1.10

x  1  iy
The function f  z   , is analytic in ,except at z  1 ,
 x  1
2
 y2
Since on using (3.1.3), we have

zz zz
1  i z 1 1
f  z  2 2i  
2 2 zz  z  z  1 z 1
 zz   zz
  1   
 2   2i 

i.e. f  z  is analytic except at z=1

Example 3.1.11
53

Consider f  z   x2  y 2  3x  5  5iy , then on using ( 3.1.4 ) above we obtain

 zz
f  z   
   3 z  z   5  i3  z  z   zz  3z  5
2 zz 2
     
 2  4i 2  2   2i 

Which is nowhere analytic (i.e f  z  is not analytic).


This is so due to the presence of z which is itself not analytic.

Which is nowhere analytic (i.e. f  z  is not analytic).


ACTIVITY
This is so due to the presence ofSelf
z which is itself3.1.12
test exercise not analytic.

1) Following
Use theorem the worked outfind
3.1.6 to examples above, let
the derivatives of us
thework out thefunctions
following following
exercise below in5a very similar manner.
a) f  z    z 2  2i 
b) g  z   6z 3  8z 2  2iz 10
z2  9
c) h z 
z 3  32
2) Determine the points at which the function f  z  fails to be analytic
1
a) f  z 
z  2  3i
iz 3  3z
b) f  z  2
z 1
2z 1
c) f z  2
z  z6

In the next section we consider criteria for determining if a given function is


analytic or not

3) Determine if the following functions are analytic or not


54

3.2 THE CAUCHY-RIEMANN EQUATIONS

If z  x  iy and w  f  z   u  x, y   iv  x, y  , then at each point where


f  z  is differentiable we have.
u  v , u   v (3.2.1)
x y y x
These equations are often referred to as the Cauchy-Riemann equations

Theorem 3.2.2

A necessary condition for a function f  z   u  x, y   iv  x, y  to be


differentiable at a point z0 is that the Cauchy-Riemann equations hold at z0 .
Consequently if f  z  is analytic in an open set, then the Cauchy-Riemann
equations must hold at every point of the domain D.

Example 3.2.3

Let f  z    x 2  y   i  y 2  x  , then u  x, y   x2  y and v  x, y   y 2  x .And


u v
since  2x ,  2y
x y
u v
1,  1
y x
Hence the Cauchy-Riemann equations are satisfied only if we allow x=y, i.e.
only on the line x=y and which is therefore not an open disc.
Thus by theorem (3.2.2) f  z  is nowhere analytic.

Theorem 3.2.4

A necessary and sufficient condition for f  z   u  x, y   iv  x, y  to be


analytic in a region D is that the Cauchy-Riemann equations (3.2.1) i.e.
u v u v
 ,  are continuous in D.
x y y x
In other words f  z  is analytic in D if and only if the Cauchy-Riemann
equations are satisfied in D, provided that these partial derivatives are
continuous in D.
55

Example 3.2.5

In this example we show that if f  z  is analytic then the Cauchy-Riemann


equations are satisfied.
2
Let f  z   z 2   x  iy   x2  y2  2xyi
Since f /  z   2 z exists, it means according to theorem (3.2.2) or theorem
(3.2.4) the Cauchy-Riemann equations must be satisfied.
u v
Infact if we put u  x, y   x2  y 2 and v  x, y   2xy , then  2x  and
x y
u v
 2 y   and hence the assertion.
y x

Example 3.2.6

In this example we demonstrate the converse of theorem (3.2.2).For this


consider the function u  x, y   e x cos y , v  x, y   e x sin y
Both functions u and v together with their partial derivatives,
u u v v
, , , and are continuous at all points in the complex plane.
x y x y
Further more these partial derivatives satisfy the Cauchy-Riemann
conditions everywhere in the complex plane
u x u u v
i.e.  e cos y  and  e x sin y  
x y y x
and this is true for all z  x  iy  .Hence from
f  z   u  x, y   iv  x, y   e x cos y  ie x sin y , and on using (3.2.2) above we
see that f /  z  exists and furthermore
u v
f /  z    i  e x cos y  ie x sin y  f  z 
x x
Hence f  z  is analytic for all z

Example 3.2.7

We know that f  z   z 3  1 is analytic on the entire plane. Verify the Cauchy-


Riemann equations
56

Solution

Let f  z   u  x, y   iv  x, y  with z  x  iy ,then


f  z   z3  1  x3  3xy 2  1  i 3x2 y  y3 
Now put u  x, y   x3  3xy 2 1 and v  x, y   3x2 y  y3
u u v v
Then  3x2  3 y 2 and  6 xy and  3x2  3 y 2 and  6 xy
x y y x
u v u v
Hence  , and   , and so the Cauchy-Riemann equations are
x y y x
satisfied.

Example 3.2.8

z3  2z  1
Let f  z   .Find the set of points where f  z  is analytic and then
z3  1
compute its derivative.

Solution

It is clear that f  z  would not be differentiable (an hence not analytic) at the
points where the denominator, z3 1  0 . Thus in describing the set of points
at which f  z  is analytic, we must avoid points at which z3 1  0 , i.e. the
i 5 i
 i
points z  e , e and e 3
3

 
Hence f  z  is analytic on the set of points A  z  : z3 1  0 i.e. the
points which are the cube roots of -1.

Now
3
 2
 2 3

df d  z3  2 z  1  z  1 3z  2  3z z  2 z  1
 
 
 
dz dz  z3  1  2
  3
z 1

df 2  4 z3
i.e  ,
 
dz 2
z3  1
57

Example 3.2.9

Show that f  z   z is not analytic


Solution
Let z  x  iy , so that z  x  iy . Then f  z   z  x  iy  u  x, y   iv  x, y 
Where u  x, y   x and v  x, y    y
u v u v
But  1 and  1, hence 
x y x y
So that the Cauchy-Riemann equations are not satisfied. Thus f  z   z is
not analytic.

Example 3.2.10

Show that the function f  z   x2 y  ix is continuous everywhere in the Z -


plane but is nowhere analytic.

Solution
Let u  x, y   x2 y , and v  x, y   x . Then following from the continuity in
real analysis we can see that u  x, y  and v  x, y  are continuous for all x and
y.Hence f  z  is continuous everywhere in the Z-plane
On the other hand, since f  z   u  x, y   iv  x, y  = x 2  ix with u  x, y   x2 y
u u v v
and v  x, y   x , we have  2 xy ,  x2 ,  1 , and 0
x y x y
Then the Cauchy-Riemann equations
u v u v
 ,   , are not satisfied, since this would mean 2 xy  0 and
x y y x
x2  1,which have no real solutions for x and y
Hence f  z  is nowhere differentiable and therefore nowhere analytic.

You have had enough examples on this concept and therefore, you should be
in a position to answer the following questions:
58

ACTIVITY
Self –test exercise 3.2.11
1. Use the Cauchy-Riemann equations to show
that the following functions are nowhere
differentiable
a) f z  z
b) f  z   Re  z 
c) f  z   2 y  ix
2. Show that f  z   x3  3xy 2  3x  i  y 3  3x 2 y  3 y 
is differentiable on the coordinate axes but is
nowhere analytic
3. Verify that the real and imaginary parts of the
following functions satisfy the Cauchy-
Riemann equations and so deduce the
analyticity of each
a) f  z   z 2  5iz  3  i
b) g  z   ze z
c) h  z   sin 2 z
4. Show that the following functions are nowhere
analytic
a) f  z   xy  iy
b) f  z   e z  cos x  i sin y 

NB/ Note that, using the Cauchy-Riemann conditions we can always


construct an analytic function f  z  , if we know its real part u  x, y  or
imaginary part v  x, y  and the value f  z0  so that
f  z   u  x, y   iv  x, y  is analytic

Example 3.2.12

Find the analytic function w  f  z  , from its known real part


u  x, y   2e x cos y and an additional condition that f  0  2
59

Solution
u
We have  2e x cos y . The first Cauchy-Riemann conditions
x
u v v
implies that  , so that  2e x cos y . This means that
x y y
v  x, y    2e cos ydy  2e sin y    x  , where   x  is yet to be
x x
found.
Finding the derivative of v  x, y  with respect to x and using the
second Cauchy-Riemann conditions we obtain:
u
2e x sin y   /  x     2e x sin y , which implies that  /  x   0 , and
y
hence   x   c ,where c is a constant. Thus v  x, y   2e x sin y  c , and
therefore:

f  z   2e x cos y  i  2e x sin y  c   2e x cos y  2ie x sin y  ic  2e x  cos y  i sin y   ic


 2e x eiy  ic  2e z  ic

c can be found from the condition that f  0  2 , i.e 2e0  ic  2 or c=0


The answer is f  z   2ez

Example 3.2.13

Find the analytic function w  f  z  if its imaginary part is


v  x, y   3x  2 xy and if f  i   2
Solution.
CRE:
u  v , u   v
x y y x
According to first condition, we obtain
 
u  2 x  u x, y  x2  y
x     y  ?

According to the second equation, we obtain

 /  y   3  2 y    y   3 y  y 2  c
60

Hence w  f  z   u  x, y   iv  x, y   x2  3 y  y 2  c  3x  2xy  i

But f  i   2 i.e when x  0 and y  1 then f  z   2

Meaning 2  3 1 c  c  0

 f  z   x 2  3 y  y 2   3x  2 xy  i  x 2  2 xyi  y 2  3i  iy  x    x  iy   3i  x  iy   z 2  3iz


2

In similar manner to the one explained above answer the given exercise:

ACTIVITY

Self help exercise 3.2.14


Reconstruct the function f  z  analytic in a neighborhood of zo from the
known real part u  x, y  or imaginary part v  x, y  and the value f  z0 
a) v  x, y   3x  2 xy , f  i   2

b) u  x, y   x2  y 2  2x , f  i   2i  1

c) u  x, y   x  e x cos y , f  0  1

d) v  x, y   2sin 2 x sinh 2 y  y , f  0  2

3.3: HARMONIC FUNCTIONS

Definition 3.3.1

A function   x, y  is said to be harmonic in a domain D if in this domain it


has continuous partial derivatives up to the second order inclusive and
 2  2
satisfies Lap laces equation  0
x 2 y 2
61

If a function f  z   u  x, y   iv  x, y  is analytic in a domain D, both its real


part u  x, y  and imaginary part v  x, y  are harmonic in D. However, if
u1  x, y  and v1  x, y  are two harmonic functions, f1  x, y   u1  x, y   iv1  x, y 
may not be analytic: The analyticity of f1  z  requires that u1 and v1 satisfies
the Cauchy-Riemann conditions.
Two harmonic functions that satisfy the Cauchy-Riemann conditions are
said to be conjugate harmonic.

Example 3.3.2

The following problem contains pairs of harmonic functions u and v.Find


among then the pairs of conjugate harmonic functions.
a) u  3  x2  y 2  , v  3x 2 y  y 3
b) u  x, v  y
c) u  e x cos y  1 , v  1  e x sin y

Solution

a) Given u  3  x 2  y 2  and v  3x 2 y  y 3
u  2u
 6x ; 6
x x 2

u  2u
 6 y ;  6
y y 2

 2u  2u
Then
x y
2  
 2  0  u  3 x 2  y 2 ,is harmonic

v  2v
 6 xy ;  6y
x x 2

v  2v  2v  2v
 3x 2  3 y 2 ,  6 y,  2  0  v  3x 2 y  y 3 is
y y 2 x y
2

harmonic.

u v u v
For the Cauchy-Riemann equations:  , 
x y y x
62

u v u v
Here  6 x and  3x 2  3 y 2   i.e the first condition is not
x y x y
satisfied and therefore u  3  x 2  y 2  and v  3x 2 y  y 3 are not a
harmonic conjugate pair.

In text question

In a very similar manner you can attempt (b) and(c).

Again iam convinced that the steps indicated above are quite clear and
hence you can be able to answer the questions here below:

ACTIVITY
EXERCISE 3.3.3
1. Show that the following functions are harmonic a)
u  x 2  2 x  y 2 b) u  2e x cos y c) u 
x
x  y2
2
d) 
u  ln x 2  y 2 
2. Can each of the following function be the real or imaginary
part of an analytic function f  z   u  x, y   iv  x, y 
a) u  x2  y 2  2 xy b) v  ln  x 2  y 2  c) v  
1 2
2
x  1 y 2

3. When is the trinomial u  ax 2  2bxy  cy 2 , a harmonic function?


a,b and c are real numbers.
4. The following problems contain pairs of a harmonic function,u
and v.
Find among them pairs of conjugate harmonic functions
a) u  3  x 2  y 2  , v  3x 2 y  y 3
x y
b) u , v
x  y2
2
x  y2
2
63

3.3: HARMONIC FUNCTIONS

Definition 3.3.1

A function   x, y  is said to be harmonic in a domain D if in this domain it


has continuous partial derivatives up to the second order inclusive and
 2  2
satisfies Lap laces equation  0
x 2 y 2
If a function f  z   u  x, y   iv  x, y  is analytic in a domain D, both its real
part u  x, y  and imaginary part v  x, y  are harmonic in D. However, if
u1  x, y  and v1  x, y  are two harmonic functions, f1  x, y   u1  x, y   iv1  x, y 
may not be analytic: The analyticity of f1  z  requires that u1 and v1 satisfies
the Cauchy-Riemann conditions.
Two harmonic functions that satisfy the Cauchy-Riemann conditions are
said to be conjugate harmonic.

Example 3.3.2

The following problem contains pairs of harmonic functions u and v.Find


among then the pairs of conjugate harmonic functions.
a) u  3  x2  y 2  , v  3x 2 y  y 3
b) u  x, v  y
c) u  e x cos y  1 , v  1  e x sin y

Solution

a) Given u  3  x 2  y 2  and v  3x 2 y  y 3
u  2u
 6x ; 6
x x 2

u  2u
 6 y ;  6
y y 2

 2u  2u
Then 
x 2 y 2
 
 0  u  3 x 2  y 2 ,is harmonic
64

v  2v
 6 xy ;  6y
x x 2

v  2v  2v  2v
 3x 2  3 y 2 ,  6 y,   0  v  3x 2 y  y 3 is
y y 2 x 2 y 2
harmonic.

u v u v
For the Cauchy-Riemann equations:  , 
x y y x

u v u v
Here  6 x and  3x 2  3 y 2   i.e the first condition is not
x y x y
satisfied and therefore u  3  x 2  y 2  and v  3x 2 y  y 3 are not a harmonic
conjugate pair.

In a very similar manner you can attempt (b) and (c)


65

ACTIVITY
EXERCISE 3.3.3

1. Show that the following functions are harmonic


a) u  x 2  2 x  y 2

b) u  2e x cos y

x
c) u
x  y2
2

d) u  ln  x 2  y 2 
2. Can each of the following function be the real or imaginary
part of an analytic function f  z   u  x, y   iv  x, y 
a) u  x2  y 2  2 xy

b) v  ln  x 2  y 2 

c) v  
1 2
2
x  1 y 2

3. When is the trinomial u  ax 2  2bxy  cy 2 , a harmonic function?


a,b and c are real numbers.

4. The following problems contain pairs of a harmonic function,u


and v.
Find among them pairs of conjugate harmonic functions
a) u  3  x 2  y 2  , v  3x 2 y  y 3
x y
b) u , v
x  y2
2
x  y2
2
66

LECTURE FOUR:

INTEGRATING FUNCTIONS OF A COMPLEX VARIABLE

4.0 INTRODUCTION

One of the most important theorems in the study of calculus is the


fundamental theorem of integral calculus because it shows the relationship
between integration and differentiation and also gives a method for
evaluating integrals. Our aim in this lecture is to look for a complex analog
of this theorem. We will see that unlike the real-valued case, complex
integration between any two points in the complex plane can be carried
out along an infinite number of paths between two points. However, the
required analog, namely ‘Cauchy’s integral theorem, which is the
fundamental theorem of complex integration, says that for analytic
functions, complex integration is not dependent on any of the many paths
joining two points.

OBJECTIVES

By the end of this lecture, you should be able to:


a) Prove Cauchy’s theorem.
b) Use this theorem to evaluate simple integrals.
c) Demonstrate by way of examples the property of
independent path
67

4.1 SIMPLY AND MULTIPLY CONNECTED REGIONS

Definition 4.1.1

A region  is said to be simply connected if any simple closed curve which


lies in  can be shrunk to a point without leaving the region 

Example 4.1.2

Let  be the region defined by z  2 .If  is any simple closed curve lying
in  ,we see that it can be shrunk to a point which lies in  and this does not
leave  ,so that  is simply connected.
Y

Fig 3
Z 2

X

Definition 4.1.3

A region  which is not simply connected is called multiply connected


region.

Example 4.1.4

If  is the region defined by 1  z  2 ,then there is a simple closed curve 


lying in  which cannot be shrunk to a point without leaving  , so that 
is multiply connected region.
68

Z 2 Fig 4
Z 1


X

A simply connected region is one which does not have any “holes” in it
while a multiply- connected region is one which does.

4.2 CONVENTION REGARDING THE TRAVERSAL OF A CLOSED


PATH
The boundary C of a region is said to be traversed in the positive sense of
direction if an observer traveling in this direction (and perpendicular to the
plane) has the region to the left. This convention leads to the direction
indicated by arrows in the above figures.

We use the special symbol  f  z  dz to denote integration of f  z  around


C

the boundary C in the positive sense.


The integral around C is often called a contour integral  f  z dz
C
4.3 CONNECTION BETWEEN REAL AND COMPLEX LINE
INTEGRAL

Suppose that a single-valued function f  z  is defined and continuous in a


domain D and that C is a piecewise smooth, closed or open oriented curve in
D.
69

Also suppose that z  x  iy and f  z   u  x, y   iv  x, y  ,where u and v are


real-valued functions of x and y.

Calculating the integral of a function f  z  of complex variable z reduces to


calculating ordinary line integral, namely

 u  iv   dx  idy   udx  uidy  ivdx  vdy  udx  vdy  i  vdx  udy 

 f  z  dz   udx  vdy  i  vdx  udy (4.3.1)


C C C

The integral  f  z  dz
C
generally depends on the path of integration. This is

some times taken as a definition of a complex line integral.

Example 4.3.1

Evaluate the integral  1  i  2 z  dz , along the following curves, connecting


C

points z1  0 and z2  1  i

a) The straight line connecting z1 and z2

b) The parabola y  x 2

c) The broken line z z z with z3  1


132
Solution

We write the integrand In the form 1  i  2 z  1  2 x   i 1  2 y 


Here u  1  2x and v  1  2 y
Applying (4.3.1) we obtain

 1  i  2 z dz   1  2 x  dx  1  2 y dy  i  1  2 y  dx  1  2 x  dy
C C C

a) The equation of the straight line passing through the points


70

z1  0 , and z2  1  i is y  x, 0  x  1 which means that dy  dx and


whence
 1  i  2 z dz   1  2 x   1  2 x  dx  i  1  2 x   1  2 x  dx = 2 i  1
C C C

1 1 1
=  4 xdx  i  2dx  (2 x2  2ix)0  2  2i
0 0

b) For the parabola y  x 2 , we have dy  2 xdx, 0  x  1 , consequently

 
1 1

  4
1  I  2 z dz   1  2 x   1  2 x 2 2 x  dx  i  1  2 x 2  1  2 x  2 x  dx = 2  i
3
C 0 0

c) On the segment z1 z3 , we have y  0 and dy  0, 0  x  1.


On the segment z3 z2 , we have x  1 , and dx  0, 0  y 1
Employing the linearity of line integrals, we obtain
 1  i  2 z dz   1  i  2 z dz   1  i  2 z dz 
C
z1 z3 z3 z2

1 1 1 1

 1  2 x  dx  i  dx   1  2 y  dy  i  1  2.1 dy  2
0 0 0 0

TAKE NOTE
The above example shows that the integral of a continuous
but not analytic function generally depends on the shape of
the integration path
71

For a clear understanding of the meaning of the above remark I recommend


that you attempt the exercise below

ACTIVITY 4.3.3

In manner similar to the one explained above,


 2,5
1.Evaluate  3x  y dx   2 y  x  dy ,along
 0,1
a) The curve y  x 2  1
b) The straight line (0,1) to (2,5)
c) Straight line (0,1) to (0,5) and then (0,5) to (2,5)

2.Evaluate the following integrals:


a)  e z Re zdz , C is the straight line connecting points z1  0 and
2

z2  1  i

 e dz , C: (i) the arc of the parabola y  x 2 connecting points


z
b)
C

z1  0 and z2  1  i
(ii) the segment of the straight line connecting these
points
72

Note the following special cases

a) If f  z  is analytic in a simply connected domain D, the value of the


integral does not depend on the path of integration. In this case
 f  z  dz  0
L

Where L is any piece-wise smooth, closed curve in D

b) If curve C is given parametrically i.e x  x  t  , y  y  t  and the


beginning and end of curve C corresponds to t  t0 and t  t1
respectively, then
t1

 f  z  dz   f  z t z t  dt
/
, where z t   x t   iy t  (4.3.4)
C t0

Example 4.3.5

Evaluate the integral


 e dz , where C is the segment of the straight line y=-x, connecting points
z

z1  0 to z2    i

Solution

The parametric equations of C are x=t , y=-t


Or in complex form z  t  it  dz  1  i  dt ,where the real variable t varies
between 0 and  .
Applying (4.3.5) above, we obtain

1  i t it
 

 e dz   e 1  i  dt  1  i   e dt 

z t it t it
e  e  1 i
1 i
0 0
C 0

c) If the function f  z  is analytic in a simply connected domain D


containing points z0 and z1 ,then Newton-Leibnitz formula is valid:
73

z1

 f  z  dz    z     z     z 
z1
1 2 z0 (4.3.6)
z0

Where   z  is antiderivative of f  z  ,  /  z   f  z  in D

Example 4.3.7
2i

  3z 
 2 z dz
2
Evaluate the integral
1i

Solution

Since the integrand f  z   3z 2  2 z is analytic everywhere, applying Newton-


Leibnitz formula (4.3.6), above we find that
2i

  3z      2  i    2  i   1  i   1  i   7  19i
2 i 3 2 3 2
2
 2 z dz  z 3  z 2 1i
1i

d) If two functions say f  z  and g  z  are analytic in a simply connected


domain D and if z0 and z1 are any two points in D, then the following
integration by parts formula is valid:

z1 z1

 f  z  g /  z  dz   f  z  g  z   z1
z0   g  z  f /  z  dz
z0 z0

Example 4.3.8
i
Evaluate the integral  z cos zdz
0

Solution

The function f  z   z and g  z   cos z are analytic in the entire z plane.


74

Integrating by parts we obtain


1 e
i i i

 z cos zdz   z  sin z  dz  z sin z 0  sin zdz = i sin i  cos z 0   sinh1  cosh1  1 
/ i i

0 0 0
e

e) If the path of integration is a ray starting at point z0 or a circle centered at


z0 ,it is expedient to introduce the change of variables
z  z0   ei

Example 4.3.9
Evaluate the integral   z 2  z z dz , where C is a an-arc of the circle z  1 ,
C

0  arg z  

Solution
We put z  ei , then dz  iei d , and

 

z  1 
2
 zz dz   ie i
e i 2
  
 1 d  i  ei 3  ei d   ei 3  ei  0  
3 
8
3
C 0 0

To confirm that you understood the examples illustrated above, here is an

exercise for you to do:


75

ACTIVITY 4.3.10

Evaluate the integral


 2,4

  2 y  x  dx  3x  y  dy , along
2
1.
 0,3

a) The parabola x  2t , y  t2  3
b) a straight line from (0,3) to (2,4)
2. Evaluate  zdz , from z  0 to z  4  2i
C

Along the curve C given by z  t 2  it


1i
3. Evaluate   2 z  1 dz
1 i

4.  Re zdz , C: z  2  it , 0  t 1
C

4.4 GREEN’S THEOREM IN THE PLANE

Let P( x, y) and Q( x, y) have continuous partial derivatives in a region


 and its boundary C. Green`s theorem states that

 Q P 
 C
Pdx  Q dy   
  x

y
dx dy
 (4.4.1)

The theorem is valid for both simply and multiply connected regions.

Example: 4.4.2

Evaluate  5x  6 y  3dx  (3x  4 y  2)dy.


C

around a triangle in the xy plane with vertices at (0, 0), (4, 0) and (4,3).
76

Y
(4,3)
Fig 5

X
(4,0)
(0,0)
Solution:

According to Green’s theorem (4.4.1)

 Q P 
 c
Pdx  Qdy   
 
 dx dy.
x y 
where
P  5x  6 y  3  p  6
y

Q  3x  4 y  2  Q  3.
x
Therefore

 (5x  6 y  3)dx  (3x  4 y  2)dy   (3  6)dx dy 


C 

4 3   4
= (3)  dx dy  (3)    dy dx   (3)(3)  dx  36
   
 
 0  0   o

This is the integral around the square, but we need around a triangle, i.e.
(36)  2  18,
77

4.4.4 Example:

Verify Green`s Theorem in the plane for

 2  2 3
 c  x  2 xy dx  ( y  x y)dy
where C is a square with vertices at (0, 0), (2, 0), (2, 2), (0, 2).

Solution

According to Green’s theorem, we have

 Q P 
c Pdx  Qdy    x  y dx dy.
 
Where

P  x2  2 xy  P  2 x
y

Q  y2  x3 y  Q  3x2 y
x
Therefore

 Q P  22  2 
c Pdx  Qdy  
  x y dx dy.     3x y  2 y  dxdy 
  00

22   
 2 y  2 y dxdy  2  2  3x2 y  2 y  dx dy 
  3 x     
00  
0  0  

2 3 2 dy  2 4 ydy  2 y 2 2  8
  x y  2 xy 
0 0  0 0
78

4.4.5. Example
Verify Green`s Theorem in the plane for
 2 2
C  2 xy  x dx  ( x  y )dy
Where C is the closed curve of the region bounded by y  x2and y2  x
Solution
Method 1:
The plane curves y  x2and y2  x intersects at (0, 0) and (1, 1)

Y
y  x2

y2  x
I2
Fig 6
(1, 1)

 I1
X
(0,0)

I  I1  I 2

Along y  x2  dy  2 xdx and 0  x  1 . we have

1  2x  x2  x2 dx  ( x  x4 )2xdx  7
I  o
1  
  6
Along

y 2  x  dx  2 ydy.
I  1o  2 y 2. y  y 4 2 ydy  ( y 2  y 2 )dy   17
2   15
7 17 1
Therefore the required integral   
6 15 30
79

Method 2. Using Green`s theorem.


 Q P 
  x  y dxdy    2 xy  x dx  ( x  y
2 2
)dy
C

 x : 0  1 y  x 2

 y : x  x, y  x
2

 
  1  2 x dxdy    1  2x  dy dx
1 x 1 x
= o x 2
o x 2

1  12 1
2 1
= o  x  2 x.x 2  2 x.x dx  30
Hence Green`s theorem is verified.

4.5 CAUCHY`S THEOREM

Let f ( z ) be analytic in a region  and on its boundary C. then


 C
f ( z )dz  0
It was proved by use of Green`s theorem with added restriction that f 1 ( z ) be
continuous in  . However Goursat gave a proof which removed this
restriction. For this reason, the theorem is sometimes called the Cauchy-
Goursat theorem.
Theorem Cauchy  Goursat 
Let f ( z ) be analytic in a region bounded by two simple closed curves C and
C1 (where C1 lies insider C). Then

 C
f ( z )dz   C1
f ( z )dz
(4.5.1)

Fig 7


C
80

The result shown that if we wish to integrate f ( z ) along curve C1 , we can


equivalently replace C by any circle C1 so long as f ( z ) is analytic in the
region between C and C1 .

Example 4.5.2

Prove Cauchy`s theorem  C


f ( z )dz  0

Proof:

if f ( z )  u  iv is analytic then the Cauchy-Riemann equations are satisfied


u v u v
i.e.  ; 
x y y x
Thus. If a function f  z  is analytic throughout a simply connected domain
D, then  C
f ( z)dz  0 for every closed contour C lying on D.

 C   u  iv  dx  idy 
f ( z )dz 
C

  udx  vdy  i   vdx  udy 


C C
uP 2nd P  v
(Applying Green`s theorem) 1st
v  Q Qu
 Q P 
 Pdx  Qdy    x  y dxdy
C 

 v u   u v 
     dxdy  i    dxdy
 
x y   
x y 

0  0 0
2nd CRE 1st CRE

v u u v
sin ce  and 
x y x y

4.5.3: Example:

Prove
81

(a)  dz  0 ( f ( z )  1)
(b) C zdz  0 ( f ( z)  z)
(c) C ( z  zo )dz  0 ( f ( z)  z  zo )
Where C is any simple closed curve and zo is a constant
solution
This follows at once from Cauchy`s theorem,
Since the functions 1, z and ( z  zo ) are analytic inside C and have
continuous derivatives.

Suppose Z o is inside C and let  be a circle at zo so that  is inside C.Then


 C
dz   
dz

 : z  zo  r
z  zo  rei
Equation of a circle in exponential form
z  rei  z0 , 0    2
dz  irei d
2u ei
 dz   irei d  ir 2
So o  rei 2  reio
 o i
= r  ei 2  1
= r cos 2  i sin 2 1
= r (0)  0

z  rei
z  z0  rei
(b)  C
z dz   
z dz. 0    2
 z  zo  rei
dz  irei d
z  rei irei d
2
 
zdz  
o o
2 2
= o
zoir ei d   r 2i 2ei d
o
2 2
= izo r  e d  r i i 2 2
 ei d  0
o o
2
(c)   z  z dz    z  z dz  
C o  o o
r ei irei d  o
82

z  zo  rei
2z
dz  irei   ir 2 ei 2 d
o
2
 ir 2  ei 2 d
o
2
 e i 2  ir 2 i 4 ir 2
 ir 
2
   e  e i0
   cos 4  i sin 4  1
 2i  0 2i 2
r2
  0  0
2

4.6 CAUCHY INTEGRAL FORMULAS

If f ( z ) is analytic inside and on a simple closed curve C, and “a” is any


point inside C, then the value of the function at point “a” is given by
f (a)  1 C f ( z)dz    dz  2 i  f  a 
f z
2 i z  a C z a
(4.6.1)

Fig 8

.a C

The nth derivative of f ( z ) at z  a is given by the formula

f  z 2 i  f  n   a 
f   (a)  n! C f ( z) dz  
n
dz 
2 i ( z  a)n1 n 1
C  z a 
n!

(4.6.2)

The result (4.6.1) can be considered a special case of (4.6.2) with n=0, if we
define 0!=1.
83

The result (4.6.1) and (4.6.2) are called Cauchy Integral formulas and are
quite remarkable because they show that if a function f ( z ) is known on the
simple closed curve C, then the values of the function and all its derivatives
can be found at all points inside C.

Example:4.6.3

Prove Cauchy`s integral formula,


1 f ( z)
f (a ) 
2 i  C za
dz
(4.6.1)

Proof:

f  z
the function is analytic inside and on C except at the point z=a
za

Fig 9
 

f  z  dz f  z  dz
We have 
C
za


za
Where we can choose  as a circle of radius  with centre at a. then an
equation for  is
z  a   or z  a   ei where 0    2

2 f (a   e )i e d
i i
f ( z )dz
  za

o  ei
2
 i f (a   ei )d
o
f ( z) 2
i.e.  C za
dz  i  f (a   ei )d
o
(4.6.3
Taking the limit of both sides of (4.6.3) we have
f ( z )dz 2
 C za
 lim i  f (a   ei )d
 0 o
84

2
= i o lim f (a   ei )d
2
= i o f (a)d
= 2 i f (a).
f ( z)
Or  C za
dz  2 i f (a )

1 f ( z)
 f (a ) 
2 i  C za
dz

Example 4.6.4

sin  z 2  cos  z 2
Evaluate: c ( z  2)( z  1) dz.

Solution:
1 1 1
  , we have
( z  2)( z  1) z  2 z  1

sin  z 2  cos  z 2 sin  z 2  cos  z 2 dz sin  z 2  cos  z 2 dz


C ( z  2)( z 1) dz  C z2
 C z 1
= I1  I 2
Cauchy`s integral formula gives
I1  2 i f (2)  2 i sin 4  cos 4   2 i
I 2  2 i f (1)  2 i sin   cos    2 i

sin  z 2  cos  z 2
Therefore C ( z  2)( z 1) dz  I1  I 2  2 i  2 i  4 i
NB:
f ( z)
 C z  zo
dz  2 i f ( zo )

Example 4.6.5

Evaluate
e2 z
c ( z  1)4 dz, where C is the circle z  3
85

Solution
4  n 1  n  3
z  a  z  1  a  1, f ( z )  e 2 z
n! f ( z)
Using the formula f ( n ) (a)  
2 i ( z  a)41
dz

So f 1 ( z )  2e2 z , f ( z )  4e2 z , f ( z )  8e2 z


3! f ( z)
Hence 8e2  
2 i C ( z  1)4
dz 

e2 z 8e2 8e2 2 i 8 i
C ( z  1)4 3!
 2 i 
1.2.3.
 2.
3e

With the aid of the examples above make sure that the following questions
are correctly answered

Exercise 4.6.6
Evaluate the following integrals employing Cauchy`s
Integral formula (all circles are circuited,
counterclockwise)
z
sin( )
ez
1.(i)  z 1 2
z  2z
dz (ii)  z 1 2 z 2  2 z  3
2 dz

2.Employing Cauchy’s integral formula, evaluate


2
ez
the integral  2 dz with
C
z  6z
(i) C : Z  2 1 C : z  2  3 C : z  2  5
3.Evaluate the following integrals by employing
Cauchy’s integral formula (all circles are circuited
counterclockwise):
dz
(i) 
z 5
z  16
2

eiz
(ii)  z 2  1dz
z i 1
86

LECTURE 5

POWER SERIES, TAYLOR SERIES, LAURENT SERIES IN THE


COMPLEX DOMAIN

5.0 INTRODUCTION

Power series are the most important type of series in complex analysis. The
reason is that they represent analytic functions and, conversely, every
analytic function has power series representations, called Taylor series.
These Taylor series are the complex analogues of the Taylor series in real
calculus. Indeed, if we replace the real variable in the latter series by a
complex variable, we may ‘’extend’’ or ‘’continue’’real functions to the
complex domain. We shall end with the representation of analytic functions
by Laurent series. These are series involving positive and negative integral
powers of the independent variable. They are also useful for evaluating
complex and real integrals as we shall see later.

Objectives
At the end of this lecture you should be able to
a) State Abel’s theorem
b) Find radius of convergence of a power series
c) Expand a given function into a Taylor series
about any point
d) Find circles of convergence
e) Expand a function into Laurent series
87

Perhaps we are now ready to define power series and to consider in


particular the Taylor and Laurent series

Definitions and convergence tests 5.1

Suppose we have a series with complete valued terms



z1  z2  ...zn  ...   zn
n 1 (5.1.1)
Where zn  xn  i yn

The series (5.1.1) is said to converge if both


 

X
1
n (5.1.2) and y
1
n (5.1.3)

converge

The series (5.1.1.1) is said to be absolutely convergent if z
1
n (5.1.4)
converge.

The series (5.1.2) and (5.1.3) and (5.1.4) are series with real terms and their
convergence is studied by applying the known convergence tests in the real
domain
A necessary condition for the convergence of series (5.1.1) is that
lim z  0
n n
5.1 Power series and radius of convergence

A series of the type Co  C1 z  C2 z 2  ..., Cn z n  ...   Cn z n
n 0 (5.1.5)

Where C0 , C1 , C2 ,... are complex constants and z is the complex variable,


is called a power series in the complex domain

Abel`s Theorem 5.1.6

if the power series (5.1.5) converges for some value of z  zo , it is


(absolutely) convergent for all values of Z such that z  zo
88

If the series (5.1.5) diverges at z  z1 then it is divergent for any values


of z such that
z  z1
The domain of convergence of (5.1.5) is a circle centered at origin of
coordinates, which is called the circle of convergence of (5.1.5)
The radius of convergence R of a power series can be found by the formulas.
lim cn
R (cn  0)
n   cn 1
(5.1.7)

lim  1 
R  
n    n cn 
 (5.1.8)
Provided the limits exists.

Example 5.1.9

Find the radius of convergence of the power series



 (1 i)n z n.
n0
Solution: We find the modulus of the coefficient
cn  (1  i )n
n
cn  (1  i )  1  i  ( 2)  2
n n n 2

Employing (5.1.8) we find the radius of convergence of the given series.

lim 1 1
R 
n n n
2
22

Using the same formulas given above ,you should be in a position to find the
radius of convergence of the following given series below

ACTIVITY
EXERCISE 5.1.10
 n 
 z  cos in
(1)   
n 0  1  i 
(4) 
n 1 2n
89

5.2: Taylor and Laurent series

A single –valued function f ( z ) that is analytic at a point z  zo can be


expanded in a neighborhood of this point in the Taylor series.

f ( z )   cn ( Z  Z o ) n
n0 (5.2.1)
Whose expansion coefficient cn are given by the formulas
1 f ( z) f n ( zo )
cn 
2 i  ( z  zo )n1 dz  n! (n  0,1, 2,...) (5.2.2).

Where  is a circle centered at z  zo and lying entirely in the neighborhood


of zo at which the function f ( z ) is analytic.
The center of the circle of convergence is at zo and the edge of the circle
passes through the singular point  of f ( z ) that is nearest to zo i.e the radius
of convergence of series (5.2.1) is equal to the distance between zo and the
nearest singular point of f ( z ) .
For the function ln(1  z) and (1  z) the Taylor expansions in the
neighborhood of point zo  0 are
z 2 z3 zn
ln (1  z )  0  z    ...  (1) n 1  ... (5.2.3)
z 3 n (R=1)

(  1) 2  (  1)(  2) 3  (  1)...(  n) z n
(1  z )  1   z  z  z  ... 
2! 3! n!

(5.2.4)

For instance for   1, we have


90


1
 1  z  z 2  ...  (1) n z n  ...   (1) n z n
1 z n 0 (5.2.5)

Example 5.2.6

Expand into a Taylor series in the neighborhood of point zo  o the function


z
f ( z) 
z  2z  3
2

using the expansion (5.2.5) and find the radius of convergence of the series,

Solution:

Expand the function into partial fractions.


z 1 1 3 1
 
z  2z  3 4 z 1 4 z  3
2

We write the right hand-side in the following way:


1 1 1 1
f  z  
4 1 z 4 1 z
3
Employing the expansion (5.2.5) we obtain

1 1 z z2
f ( z)  (1  z  z 2  z 3  ...)  (1    ...)
4 4 3 9

1 4 8 28 2
 ( z  z 2  z  ...)
4 3 9 27

z 2 7
   2 z 2  3 z 3  ...
3 3 3
The singular point of the given function that is nearest to zo =0 is point z=-1
Therefore the radius of convergence of the series is R=1

Example 5.2.7

1
Expand in powers of (z-3) the function f ( z ) 
3  2 z.
1 1
Solution: f ( z)  
3  2 z 3  2( z  3  3)
91

1
=
3  2( z  3)
1 1
= 
2
3 1  ( z  3)
3
2
Substituting ( z  3) for z in (5.2.5),we obtain
3
1 1 2 22 23 
  1  ( z  3)  2 ( z  3)2  3 ( z  3)3  ...
3  2z 3 3 3 3 
1 1 2 22 23
   2 ( z  3)  3 ( z  3)  4 ( z  3)3  ...
2

3  3z 3 3 3 3
2 3
Thus series converges when ( z  3)  1 or z  3  . i.e. the radius of
3 2
3
converges is R 
2

Now its your turn to work out the following exercises in a very similar
manner to the one above

ACTIVITY:
EXERCISE 5.2.8
Expand the functions below in Taylor series using
the already established expressions.Find the radii of
convergence for these series:
a) e z in powers of (2z-1)
1
b) in powers of (z+2)
3z  1

c) sin  2z  1 in powers of (z+1)

d) ln  2  z  in powers of z


e) cos z in powers of ( z  )
4
92

Suppose now we have the series



c1 c2 c n c n

z  zo ( z  zo ) 2
 ... 
( z  zo ) n
 ...  
n 1  z  z0 
n

If c n  o and there is the finite limit


lim  c n 1 
r  
n    c n  (5.2.9)
then the series converges in the domain z  zo  r.

Example 5.2.10

Find the circle of convergence of the series



1
(i)  (1  i)
n 1
n
zn

 
n
 2 i 2
(ii) 
n 1 zn


z n
(iii) 
n 1 cos in

Solution: given the series (i)



1

1  i 
n
n 1 zn

1 1
Here C n  , C n 1. 
(1  i) n (1  i) n 1

 1 

lim  (1  i ) n 1 
lim c n 1 
Hence r   
n   c n n   1 
 (1  i ) n 
93

lim  1  1
=  
n    1  i  2

1
Therefore the circle of convergence is z 
2


(1  i )
(ii) 
n 1 zn
, C n  (1  i) n , C n 1  1  (1  i) n 1

lim 
 1  i   
n 1
lim c n1  lim
r   n 
 1 i  2
n   c n n    1  i    n  
 
Therefore the circle of convergence is z  2

The series of the type


 
c
 c  z  z    (z  z )   c (z  z )
n n n
n o n n o
n  n 1 o n o

c c
= ....   n n  ...  1  co  c1 ( z  zo )
( z  zo ) z  zo
c2 ( z  zo ) 2  ...  cn ( z  zo ) n  ...
(5.2.11)
Converges in the domain where the two series given below converge.

c n c1 c2
 (z  z )
n 1
n
 
z  zo ( z  z o ) 2
 ...
o (5.2.12)

 c (z  z )  c  c1 ( z  zo )  c2  z  zo   ....
2
n o o
n o (5.2.13)
Suppose series (5.2.12) converges in the domain z  zo  r i.e in the exterior
of the circle of radius r centered at point z  z0 and series (5.2.13) in the
circle z  z0  R .Then
1) If r>R, (5.2.11) is everywhere convergent

2) If r<R, (5.2.11) is convergent in the annulus: r  z  z0  R .Here r  0


and 0  R  

Example 5.2.14

Determine the domain of convergence of the series


94

 z  1
n
 
ein
 
 z  1
n 1
n 1 n 0 in 
2
e
Solution

ein
For the series  , we have
 z  1
n
n 1

c n  ein , and c n1  ei n1


ei n1
r  lim  lim eini in  lim ei  1
n  e in n  n 

So that the first series converges in the domain z  1  1


 z  1 , we have
n

For the series  1
n 0 in 
2
e
1
 in 
2
1 1
e
 in   i  n 1  cn
cn  e 2
and cn1  e 2
 lim  lim  1
1
n  cn 1 n   i  n 1 
 2
e

So that the second series converges in the circle z  1  1


Therefore the given series diverges everywhere

Example 5.2.15

Find the domain of convergence of the series

 3  4i     z  2i n
n


n 1  z  2i 
n  
n 0  6 

solution
 3  4i  , we have c  3  4i n ,
n

For the series    and c n1   3  4i 
n 1
n
n 1  z  2i 
n

 3  4i 
n 1

 r  lim  lim 3  4i  5 .Hence the first series converges in the


 3  3i 
n  n n 

domain
z  2i  5
95

z  2i 
 n

For the series   n


 , we have cn  6 , and cn 1  6
 n 1

n0  6 
6 n
 R  lim  6 .Hence the second series converges in the region z  2i  6 .
n  6 n1
Thus r  5  R  6
The given series therefore converges in the annulus 5  z  2i  6

In a very similar manner to the one above, below here is an activity for you
to do

ACTIVITY
SELF-TEST 5.2.16

Find the domain of convergence of the following


series
  n n
2
1)       
z
n 1  z  n 0  4 


1  zn
2) 
n 1 z
n
  n1
n 0 2

 z  i
n

3) 
n 1 2n


1 
  n  in   z  1  i 
n
4)
n 1
96

A function f  z  that is single-valued and analytic in the annulus


r  z  zo  R (including the cases where r  0 and R   ) can be expanded
in this annulus in a Laurent series:
 1 n 
     
n n
f  z    cn z  z   cn z  z   cn z  z . (5.2.17)
n 0 n 0 n0 0
Where the expansion coefficients are
1 f  z  dz
cn 
2 i  z  z 

n 1
, (n  0,  1,  2,...) (5.2.18)
0

Here  is an arbitrary circle centered at z0 and lying entirely inside the


annulus. In (5.2.17) the series
1 
c n
 cn  z  z0   
n

 z  z0 
n
n  n 1

Is called the principal part of the Laurent series, while the series

c z  z 
n
n 0
n 0

Is called the regular part of the Laurent series. For practical purposes
formula (5.2.18) for cn is usually avoided since it leads to cumbersome
calculations. The common practice is to use, when possible, the known
expansions into Taylor series of elementary functions

Example 5.2.19

Expand in Laurent series the function


1
f  z   z 2 cos   in the neighborhood of point z0  0
z
Solution
For  complex we have
2 4 6
cos   1   ... 
2!
6! 4!
1
Assuming that   , we obtain
z
1  1 1 1  1 1 1
z 2 cos  z 2 1     ...   z 2     ...
 2! z 
2 4 6
z 4! z 6! z 2
2! 4! z 6! z 4
Or
1 1 1 1
z 2 cos      z 2    ...
z
2
2 4! z 6! z 4
97

This expansion is valid for all nonzero z .In the case at hand the annulus is the entire
complex plane without one point, z  0 .The annulus is given by the following
relationship: 0  z  0   .Here r  0 , R   , and z0  0 .The given function is
analytic in the noted annulus.

Example 5.2.20
Expand in a Laurent series the function
2z  3
f  z 
z  3z  2
2

In the neighborhood of the singular points of f  z  .

Solution.

The singular points in this case are z1  1 and z2  2 .


(1) Expansion of f  z  in the neighborhood of the singular points z1  1 ,
1.e. in the annulus 0  z 1  1 . We write f  z  as a sum of partial
fractions:
2z  3 1 1
  .
z  3z  2 z  1 z  2
2

Next we write the right-hand side as


2z  3 1 1
 
z  3z  2 z  1 1   z  1
2

Applying the expansion in 5.2.5, in which   z  1 is substituted for z , we


obtain
2z  3 1
  1   z  1   z  1  ... ,
2

z  3z  2 z  1 
2 
Or
2z  3 1 
    z  1 .
n

z  3z  2 z  1 n 0
2

(2) Expansion of f  z  in the neighborhood of the singular point z3  2 ,


i.e. in the annulus 0  z  2  1. We have
2z  3 1 1 1 1 1
      1   z  2    z  2    z  2   ...,
2 3

z  3z  2 z  1 z  2 z  2 1   z  2  z  2
2

or
2z  3 1 
    1  z  2  .
n n

z  3z  2 z  2 n 0
2

In a manner similar to the worked out examples do the following exercises.


98

ACTIVITY
Exercise 5.2.21

Expand in Laurent series at z=0 the following functions:


sin z 3 z
1
1  cos z
1. 2
2. z e 3.
z z2
ez 1  cos z ez
4. 5. 6.
z z4 z3

Expand in Laurent series the following functions at the points stated


z
1. , z0  1
 z  1
2

sin z
2. , z0  2
z2
1
3. ze z i , z0  i
Expand in Laurent series the following functions in the annuli stated:

1
1. , a) 2 z 3
 z  2 z  3
b) 3  z  

1
2. , a) 0  z 1
z z
2

b) 1  z  
99

LECTURE 6:

ZEROS OF A FUNCTION AND ISOLATED SINGULARITIES

6.0 Introduction

In this lecture we make use of the Laurent expansion to classify, the


behavior of an analytic functions near its zeros and isolated singularities

OBJECTIVES

At the end of this lecture, you should be able to


a) Define the zero of a function
b) Find zeros of a function
c) Determine order of zeros
d) Define a singular point
e) Differentiate between removable, pole and essential singularity

6.1 ZEROS OF A FUNCTION

Definition 6.1.1

Suppose f  z  is a function that is analytic at say point z0 .This point is said


to be a zero of f  z  of the nth order if

f  z0   0, f /  z0   0, ...., f  n1  z0   0, f n  z0   0
100

If n=1, point z0 is said to be a simple zero of f  z 

A point z0 is a zero of a function f  z  that is analytic at z0 if and only if I a


neighborhood of this point

f  z    z  z0    z 
n

Where   z  is analytic at z0 and   z0   0

Example 6.1.2

Find the zeros of f  z   1  cos z and determine their order

Solution

Nullification of f  z  yields cos z  1, which implies that the points


zn   2n  1   n  0, 1, 2,... are the zeros of f  z  .moreover
f /  (2n  1)    sin  2n  1   0
f / /  2n  1     cos  2n  1   1  0
Which means that the zn   2n  1   n  0,  1,  2,... are second-order
zeros of f  z  .

Example 6.1.3

Find the zeros of f  z   1  ez and determine their nature


Solution

1  e z  0  zn  2n i
f /  2n i   e2n i  1  0 , which implies that the zn  2n i where n  are
simple zeros of f  z   1  ez .

In manner similar to the worked examples, make sure that the exercise
below is done correctly.
101

ACTIVITY
Exercise 6.1.4
Find the zeros of the following functions and determine their order

a) f  z   z 4  4z 2

sin z
b) f  z 
z
c) f  z   z sin z
2

6.2 ISOLATED SINGULAR POINTS


d) f  z   1  cosh z

e) f  z   cos z 3

(1  sinh z ) 2
f) f  z 
z

6.2 Isolated singular points

A point z0 is said to be an isolated singular point, or simply an isolated singularity of a


function if there is a neighborhood of this point in which the function is analytic
everywhere except at z  z0 .
An isolated singular point of a function f  z  is said to be removable if at this point
f  z  has a finite limit.
102

Example 6.2.1

ez 1
Let f  z   . The singular point of f  z  is z0  0 . Thus we
z
ez 1
have lim f  z   lim  1 . Hence, z0  0 is a removable singular point
z 0 z 0 z

A point z0 is said to be a pole of a function f  z  if f  z  tends to  as z  z0


A point z0 is a pole of a function f  z  , if and only if it is a zero of the function
1
  z 
f  z
A point z0 is said to be a pole of the nth order  n  1 of f  z  if it is a zero of the nth
1
order of   z   .When n=1, the pole is said to be simple.
f  z
  z
A point z0 is an nth order pole of a function f  z   , with   z  analytic at z0 ,
 z  z0 
n

and   z0   0

Example 6.2.2

sin z
Let f  z   .This function has two singular points, z  1 and z  1 .
z  z2  z 1
3

sin z
We take point z  1 . We write f  z   z  1 2 . Here   z  
sin z
is analytic in the
 z  1 z 1
sin1
neighborhood of point z  1 , and   1   0 . Hence, point z  1 is a second
2
order pole of the given function.
sin z
 z  1
2

Similarly, if we write f  z  in the form f  z   . We can conclude that point


z 1
z  1 is a simple pole of f  z 

A point z0 is said to be an essential singularity of a function f  z  if there is no limit for


f  z  (either finite or infinite) as z  z0 .
103

Example 6.2.3
1

Determine the type of the singular point z=0 of the function f  z   e z .


2

Solution

Let us see how this function behaves on the real and imaginary axes. On the real axis z=x
1

and f  x   e , which tends to  as x  0 .On the imaginary axis z  iy and


x2

1

f  iy   e y2
, which tend to zero as y  0 . Therefore, f  z  has no limit, either finite or
infinite, at point z=0. This means that point z=0 is an essential singularity for f  z  .

It is now your turn to demonstrate that you have understood the concepts given above by
answering correctly the following questions

ACTIVITY
Exercise 6.2.4
1. Determine the type of the singular point z0  0 of the function
1
a) f  z 
2  z  2 cosh z
2

1
b) f z 
z  sin z

sin z
c) z
e  z 1

1
d) z
e  z 1
2. Find the singular points and determine their types for the following functions
1
a)
1  sin z

1  cos z
b)
z2

z
c)
z  2z4  z3
5
104

LECTURE SEVEN: RESIDUES OF A FUNCTION

7.0 INTRODUCTION

We have seen how the study of contour integration gives great insight into
the properties of analytic functions. You will recall, for instance, the Cauchy
Goursat theorem which assures that if a function is analytic everywhere
inside and on a simple closed contour C, then the integral of the function
f  z  around C is zero. We shall show in this lecture that if however, the
function f  z  fails to be analytic at a finite number of singular points inside
C then there is a specific number, called a residue, which each of these
singular points contribute to the value of the integral of f  z  around C.

OBJECTIVES
At the end of this lecture you should be able to:
a) Define residue
b) Calculate residue
c) Evaluate given integrals using the method of residues
d) Evaluate certain real integrals

7.1 Definitions and examples

Suppose a point z0 is an isolate singular point of a function f  z  .The


residue of f  z  at a point z0 is the quantity
1
Res f  z0    f  z  dz (7.1.1)
2 i 

(Other notations are Re s f  z  , R  z0 , f  )


z  z0
105

The contour  is a circle centered at z0 and no other singular points in its


interior.
The value of the residue is the coefficient of  z  z0  in the Laurent
1

expansion of f  z  about z0 :
Re s f  z0   c1 (7.1.2)

The value of the residue at a removable singular point is zero.

If point z0 is a pole of the nth order of f  z  , then


Re s f  z0  
1 d n 1

lim n 1 f  z  z  z0 
 n  1  z  z0 dz
n
 (7.1.3)

When z0 is a simple pole (n=1),


Re s f  z0   lim  f  z  z  z0   (7.1.4)
z  z0

If in the neighborhood of point z0 the function f  z  can be written in the


form of the quotient of two analytic functions,
  z
f z 
 z
With   z0   0 ,   z0   0 , and /  z0   0 , i.e. if z0 is a simple pole
of f  z  , then
  z0 
Re s f  z0   , (7.1.5)
 /  z0 
If point z0 is an essential singularity of f  z  , then to find Re s f  z0  we must
find the coefficient c1 in the Laurent expansion of the function f  z 
about z0 ; this coefficient gives the value of Re s f  z0  .

Example 7.1.6

Find the residues of the function


sin z 2
f  z 

z3  z2
4
At its singular points,
106

Solution
sin z 2 
The singular points of are z  0 and z 
 4
z3  z2
4
At z0 ,
sin z 2 1 4
lim f  z   lim 3 lim  .
z 0 z 0 z z  0  
z
4
Therefore, point z  0 is a removable singularity of f  z  .
Hence, Re s f  0  0.

 
At point z  we have lim f  z    , i.e. point z  is a first-order pole of
4 z 4
4

f  z  .According to (7.1.4) we have


sin z 2
       sin z 2 16 2
Re s f    lim f  z   z    lim  z    lim  sin .
 4  z  4  4  z  z 3   z 2  4  z  z 3  2
16
4 4
4

Example 7.1.7

Find the residues of the function


ez
f  z 
 z  1  z  2 
3

At its singular points

Solution

The singular points of


ez
f  z 
 z  1  z  2 
3

Are z  1 and z  2
Point z  1 is a third-order pole for f  z  .According to (7.1.3)
1
We have Re s f  1  zlim
 z 2  6 z  10  e z

17
 z  2
 3
2 1 54e
Point z  2 is a first-order pole, hence by (7.1.4)
107

ez e3
Re s f  2   lim 
 z  1
z 2 3
27
Following the two examples given above, it’s your turn to find the residues
of the following functions.

ACTIVITY
Exercise 7.1.8
a) Find the residues of the function f  z    z 4  1 at its singular
1

points
 1 
b) Find the residue of the function f  z   z 3 sin  2  at the
z 
singular point of f  z 
c) Find the value of the residue at z  0 for the function
sin 3z  3sin z
f  z 
(sin z  z )sin z

d) Find the residue of the function


1

f  z   e z cos z , at point z  0
2

1
z
e
e) Find the residues of f  z   at its singular points.
1 z
108

Let us now consider the problem of evaluating a given integral  f  z  dz .


C

We recall from Laurent series that if a function f(z) is single-valued and


analytic inside and on the contour C except at some point inside C, then f(z)
has a Laurent series expansion about z0 given by

1 f  z  dz
f z   a z  z  , where an   z  z  n  0,  1,  2,...
n
,
2 i n 1
n 0
n  C 0

1
Now if n=-1, then an reduces to a1 
2 i  f  z  dz   f  z  dz  2 ia
C C
1

The coefficient a1 is often called the ‘’residue’’of f (z) at z  z0 , and denoted
by the symbol Re s f  z0   a1 .Note that a1 is the coefficient of the first terms
of the principal part of the Laurent series. Thus a1 plays a significant role in
contour integration.

7.2 THE RESIDUE THEOREM

If a function f  z  is analytic on the boundary C of a domain D and


everywhere in D except, perhaps, at a finite number of points
z1 , z2 , ... , zn , then
n

 f  z  dz  2 i  Re s f  zk  . (7.2.1)
C k 1

It follows that evaluation of such integrals depends on our ability to evaluate


residues.

7.3 Applying residues to evaluate definite integral

Example 7.3.1

ez 1
Evaluate the integral z 4 z 2  z dz
109

Solution

ez 1
In the circle z  4 the function f  z   is analytic everywhere except at
z2  z
points z=0 and z=-1.
e z _1
According to the residue theorem,  z 2  z dz  2 i  Re s f  0   Re s f  1 
z 4

Point z=0 is a removable singular point for f(z), since


ez 1
lim 1
z 0 z  z  1
Therefore Re s f  0  0.
Point z=-1 is a first-order pole, which means that
 e z  1 
Re s f  1  lim   z  1   1  e 1
z 1 z  z  1
 
ez 1
Hence,  2 dz  2 i 1  e1  .
z 4
z z

Example 7.3.2

5z  2
Evaluate  dz , where C is z  2 , taken in the positive sense.
C
z  z  1

Solution

The singularities inside C are z=0, 1. So I  2 i  k0  k1 

5z  2
Find k0 . Set g  z  
, analytic in a small circle c0 centered at 0
z 1
g  z
By the Cauchy integral formula  dz  2 ig  0  .2  2 i.k0
c0
z
5z  2
Find k1 . Set h  z   , analytic in a small circle c1 centered at 1
z
Again by the Cauchy integral formula
110

h z
 z  1dz  2 ih 1  2 i.3  2 i.k
c1
1

Hence I  2 i  2  3  10 .
Using the same arguments used in the examples above, here are some
exercises for you to do.

ACTIVITY
Exercise 7.3.3
5
a) Evaluate 
z 3
ze z dz by using the residue method

1 1
b) Find the residue of e z , at z  0 and compute  e z dz , where 0 is a
C

point within C.
1
c) Evaluate 
z 5
z 2 sin   dz
z
e z dz
d)  z 3  z  1
z 2

sin  z
e)  z2  z
dz
z 3

e z dz
f)  z4  2z2 1
z i 1
111

ANSWERS TO ALL EXERCISES

Chapter 1
Exercise 1.1.7

20 36 b a
1. (i) x , y (ii) x , y 2 2
17 17 a b
2 2
a b
2. (i) 7  5i (ii) 7  5i

5 1
(iii)   i
2 2
3. x  1, y2

Exercise 1.2.3
3
(i)   5,   arctan
4
2
(ii)   4,   
3
4
(iii)   1,   
5
(iv)   1,   2  
Exercise 1.3.6
(i) 2  cos   i sin  
  
(ii) 2  cos  i sin 
 2 2
 3 3 
(iii) 2  cos  i sin 
 4 4 
       
(iv) 2 1  sin   cos     i sin    
 2 2  2 2 
     
(v) 3 2  cos     i sin    
  4  4 
Exercise 1.3.11
     
(i) 8 cos     i sin    
  3  3 
  7   7  
(ii) 6 2 cos    i sin  
  12   12  
     
(iii) 6 3 cos    i sin   
  24   24  
112

     
(iv) 24 cos     i sin    
  6  6 
4 4
(v) cos  i sin
3 3
Exercise 1.4.4
1.
(i) 3cos 2  sin   sin 3 
(ii) cos3   3cos  sin 2 
(iii) 4sin  cos3   4 cos  sin 3 
(iv) cos 4   6 cos 2  sin 2   sin 4 
2. (i) -1 (ii) 28
1 3
3. (i)  i
2 2
1 3
(ii) 85   i 
2 2 
Exercise 1.4.8
1 i
(i) 
2
(ii) 
1
2
1  i  (iii)
1
2
 3 i 
 
3
4
(iv)  3 i (v) 1  i 
2
    3 3 
(vi)   cos  i sin  ,   cos  i sin 
 8 8  8 8 
Exercise 1.4.9

3. (i) xy  1 (ii) The hyperbola xy  1


1
(iii) The hyperbola x 2  y 2 
2
4. az  az  2c  0 , where a  A  iB
5. x2  y 2  2 y  2  0 , or x 2   y  1  3
2

This is a circle of radius R  3 centered at point (0,1)


6. zz  1  i  z  1  i  z  0
7. (i) 219 1  i 3  2 3
6
(ii)

Chapter 2

i) u  x  2 xy, v  y 2  x 2 y , (ii) u  2 x2  3 y  2 y 2 , v  4 xy  3x
113

x2  y 2 2 xy
(iii) u  2 x2  3 y  2 y 2 , v  4 xy  3x (iv) u , v
x2  y 2 x2  y 2

(v) u  1  x2  y 2 , v  2 xy (vi) u  x3  3xy 2 , v  3x 2 y  y 3

(vii) u 
x
, v
 y  1
x   y  1   y  1
2 2 2 2
x

Exercise 2.3.4
1
(i) 0 (ii) 0 (iii) (iv) -i (v) i (vi)Does not exist (vii)0
3
Exercise 2.4.5
1 1
(i) i (ii)-2i (iii)-2+11i (iv) 2 (v)-i (vi)6i (vii)0 (viii) (viiii) 
2 2

Chapter 3
Exercise 3.1.12
(1)
64 z  z 4  27 z 2
a) 10  z 2  2i  z
6
b) 18 z 2  16 z c)
z  32 
3 2

(2)
1  i 23
a) z  2  3i b) z  i c) z
2

Exercise 4.6.6

1) (i)  i (ii) i
2
 e36  1
2) (i) 0 (ii)  (iii) i
3 3
3) (i) 0 (ii)  e 1

Exercise 5.1.10

1) R  2 2) R=1 3) R 4) diverges


5) converges 6) converges

Exercise 5.2.8
114

 1 1 1 
2 1   2 z  1  2 
2 z  1   ... , R
2
a)
 2 
3
2!2 3!2
1 3 32
3 3
 5
b)  1   z  2   2  z  2   3  z  2   ... , R
2 3

5 5 5 5  3
2 z3
 sin1 2  z 1 cos1  z 1 sin1  z 1 cos1 ...
2 2 3
c)
2! 3!
R

1 z z 
2 3
z
d) ln 2     ...  , R=2
2  2 24 38 
1    1  1  
2 3

e)  
1  Z     Z     Z    ... , R
2   4  2!  4  3!  4 
1 9 41 2
f)   z  z  ... R=1
5 25 125

Self-test 5.2.16
1) 2  z  4
2) 1  z  2
3) z  i  2
4) z  1  i  1
Exercise 5.2.21

1 z z3 z5
1)     ...
z 3! 5! 7!
z 1 1
2) z3  z 2     ...
2! 3! 4! z
1 z2 z4 z6
3)     ...
2! 4! 6! 8!
1 z z2
4)  1    ...
z 2! 3!
2 1 1 z2
5) 4
 2
   ...
z 2! z 4! 6!
115

1 1 1 1 z
6)      ...
z 3 z 2 2! z 3! 4!

1 1
1. 
z  1  z  12
sin z sin z cos 2 sin 2 cos 2
  z  2   z  2   z  2   z  2   ...
2 3 4
2.
z2 2! 3! 4! 5!

 
1 i 1 1 i  1 1 i
3. 1  i    z  i     
 2! 1!  z  i
  
 3! 2!   z  i
2
 ...   
n1   n  1 !

n !
z

2n1 1   z 
 n

1. a)  n    
n1 z 3 n 0  3 

3n1  2n1
b) 
n1 zn

1 
2. a)    1 z
n n

z n 0

 1
n

b) 
n 0 z n 2

Exercise 6.1.4
a) Z=0, a second –order zero, and z1,2  2i , simple zero
b) zn  n  n  1,
 2,... simple zeros
c) Z=0, a third-order zero, and zn  n  n  1,  2,...
d) zn   2n  1  i ,  n  0,  1,  2... ,second-order zero
116


e) zn  3  2n  1 ,  n  0,  1,  2... and
4

 1 i 3
zn   2n  1 , simple zeros
2 2
i
f) zn   4n  1 ,  n  0,  1,  2... , second-order zeros
2
Exercise 6.2.4
1.
a) Z=0 is a fourth-order zero fot   z  which means that for the given
function f  z  , point z=0 is a fourth-order pole
b) A third-order pole
c) A simple pole
d) A second-order pole

2.

a) zn   4n  1 ,  n  0,  1,  2... ,a second-order pole
2
b) z=0, a removable singularity

c) Z=0, a second-order pole and z=-1, a second-order pole

Exercise 7.1.8

 3 3 
i i i i
a) z1  e , 4
z2  e 4
, z3  e 4
, z4  e 4

1 1  i 34
Re sf  z1   3   e
4z z e
i
4 4

1 1  i 94
Re sf  z2   3 3  e
4z z e
i
4 9
117

1 1 i 94
Re sf  z3   3 i
3  e
4z z e 4 4

1 1 i 34
Re sf  z3   3 i
  e
4z z e 4 4

Exercise 7.3.3

a) 25 i
b) 2 i

c)  i
3
d) 1  2e   i
1

e) 0

f)  cos1  sin1  i  sin1  cos1 
2

You might also like